Будь умным!


У вас вопросы?
У нас ответы:) SamZan.net

Внутренняя энергия вещества А 1 Внутренней энергией тела называют

Работа добавлена на сайт samzan.net: 2016-03-30

Поможем написать учебную работу

Если у вас возникли сложности с курсовой, контрольной, дипломной, рефератом, отчетом по практике, научно-исследовательской и любой другой работой - мы готовы помочь.

Предоплата всего

от 25%

Подписываем

договор

Выберите тип работы:

Скидка 25% при заказе до 20.5.2024

ПОДГОТОВКА К ЕГЭ

7. ТЕРМОДИНАМИКА

1.  Внутренняя энергия вещества

А 1

Внутренней энергией тела называют

1) кинетическую энергию хаотического движения частиц, из которых состоит тело

2) энергию взаимодействия частиц тела

3) сумму энергии хаотического движения частиц тела и энергии их взаимодействия

4) сумму кинетической и потенциальной энергии тела, движущегося на некоторой высоте над Землёй

А 2

Внутренняя энергия тела определяется

1) хаотическим движением молекул в нём и их взаимодействием

2) его движением

3) действием на него внешних сил

4) его притяжением к Земле

А 3

Внутренняя энергия 1 кг воды в запаянном сосуде определяется

1) хаотичным движением молекул и их взаимодействием

2) движением всего сосуда с водой и его притяжением к Земле

3) действием на сосуд с водой внешних сил

4) формой сосуда

А 4

От каких макроскопических параметров зависит внутренняя энергия тела?

1) только от температуры тела

2) от температуры и скорости движения тела

3) от температуры тела и расстояния от тела до поверхности Земли

4) от температуры и объёма тела

А 5

Как изменяется внутренняя энергия тела при увеличении температуры?

1) увеличивается

2) уменьшается

3) у газообразных тел увеличивается, у жидких и твёрдых тел не изменяется

4) у газообразных тел не изменяется, у жидких и твёрдых тел увеличивается

А 6

Внутренняя энергия монеты уменьшается, если её

1) охладить

2) заставить двигаться с меньшей скоростью

3) опустить к поверхности Земли

4) опустить в воду той же температуры

1

А 7

Внутренняя энергия монеты увеличивается, если её

1) нагреть

2) заставить двигаться с большей скоростью

3) поднять над поверхностью Земли

4) опустить в воду той же температуры

А 8

Внутренняя энергия гири увеличивается, если

1)

гирю поднять на 2 м

2)

гирю нагреть на 2оС

3)

увеличить скорость гири на 2 м/с

4)

подвесить гирю на пружине, которая растянется на 2 см

А 9

Совершив работу, можно изменить внутреннюю энергию

1) только газа

2) только жидкости

3) только твердого тела

4) любого тела

2. Виды теплопередачи

Теплопроводность

А 1

В металлическом стержне теплопередача осуществляется преимущественно путём

1) излучения

2) конвекции

3) теплопроводности

4) излучения и конвекции

А 2

Металлический стержень нагревают, поместив один его конец в пламя свечи. Через некоторое время температура противоположного конца  стержня повышается. Это можно объяснить передачей энергии от места нагревания  

1) в основном путём теплопроводности

2) путём конвекции и теплопроводности

3) в основном путём лучистого теплообмена

4) путём теплопроводности, конвекции и лучистого теплообмена примерно в равной мере

Конвекция

А 3

Теплообмен путём конвекции может осуществляться

1) в газах, жидкостях и твёрдых телах

2) в газах и жидкостях

3) только в газах

4) только в жидкостях

А 4  

В кастрюле с водой, поставленной на электроплиту, теплообмен между конфоркой и водой осуществляется путём

1) излучения

2) конвекции и теплопроводности

3) теплопроводности

4) излучения и теплопроводности

2

А 5

От радиатора центрального отопления воздух в комнате нагревается преимущественно за счёт

А. излучения

Б. конвекции

В. теплопроводности

Правильным является утверждение

1) только А

2) только Б

3) только В

4) А и В

А 6

На Земле в огромных масштабах осуществляется круговорот воздушных масс. Движение воздушных масс связано преимущественно с

1) теплопроводностью и излучением

2) теплопроводностью

3) излучением

4) конвекцией

Излучение

А 7

Какой вид теплообмена определяет передачу энергии от Солнца к Земле

1) в основном  конвекция

2) в основном теплопроводность

3) в основном излучение

4) как теплопроводность, так и излучение

А 8

Благодаря какому виду теплопередачи (преимущественно) в летний день нагревается вода в водоемах

1) конвекция

2) теплопроводность

3) излучение

4) конвекция и излучение

А 9

Чем объясняется смена времен года на Земле?

1) периодическими изменениями скорости вращения Земли вокруг своей оси

2) периодическими изменениями скорости движения Земли вокруг Солнца

3) отличием от 90о угла наклона оси вращения Земли вокруг своей оси к плоскости земной орбиты

4) периодическими изменениями направления движения морских течений и циклонов

А 10

В северном полушарии Земли в декабре дни короче, чем в июне, так как

1) зимой Земля движется медленнее по орбите вокруг Солнца

2) зимой Земля движется быстрее по орбите вокруг Солнца

3) в декабре ось суточного вращения Земли наклонена северным полушарием к Солнцу

4) в декабре ось суточного вращения Земли наклонена  так, что северное полушарие Земли повернуто от Солнца

А 11

Для того чтобы  человек мог существовать при температуре окружающей среды выше + 40оС, внутренние регуляторные механизмы жизнедеятельности организма человека действуют так, что:

1) между человеческим организмом и окружающей  средой при любой температуре поддерживается тепловое равновесие

2) при более высокой температуре окружающей среды увеличивается теплоотдача организма человека, а при низкой уменьшается

3) при более высокой температуре окружающей среды уменьшается теплоотдача организма человека, а при  более низкой – увеличивается

4) уровень теплоотдачи организма человека поддерживается постоянным независимо от температуры окружающей среды

А 12

Для того чтобы  человек мог существовать при температуре окружающей среды  - 40оС, внутренние регуляторные механизмы жизнедеятельности организма человека действуют так, что:

1) между человеческим организмом и окружающей  средой при любой температуре поддерживается тепловое равновесие

2) при более высокой температуре окружающей среды увеличивается теплоотдача организма человека, а при низкой уменьшается

3) при более высокой температуре окружающей среды уменьшается теплоотдача организма человека, а при  более низкой – увеличивается

4) уровень теплоотдачи организма человека поддерживается постоянным независимо от температуры окружающей среды

3. Нагревание и охлаждение вещества

А 1

При охлаждении твердого тела массой m температура тела понизилась на T. По какой из приводимых ниже формул следует рассчитывать количество отданной телом теплоты Q? с – удельная теплоемкость вещества.

  1.  с·m·T  2)       3)          4)

А 2

Удельная теплоёмкость чугуна 500 . Чугунная деталь массой 10 кг при понижении температуры на 200 К отдаёт количество теплоты равное

1) 25 МДж               2) 100 кДж                 3) 25 кДж            4) 1 МДж

А 3

Бронзовый подсвечник массой 2 кг нагрели до температуры 900 К. Какое количество теплоты выделилось при остывании подсвечника  до температуры 300 К? Удельная теплоёмкость бронзы 420 .

1) 500 кДж       2) 250 кДж

3) 750 кДж                4) 50 кДж

А 4

При передаче твердому телу массой количества теплоты, равного , температура тела повысилась на . Какое из приведенных ниже выражений определяет удельную теплоёмкость вещества этого тела?

1)                  2)                       3)                      4)

А 5

При нагревании текстолитовой пластинки массой  0,2 кг  от  30 ºC до  90 ºC потребовалось затратить 18 кДж энергии. Следовательно, удельная теплоемкость текстолита равна

1) 0,75 кДж/(кгК)

2) 1 кДж/(кгК)

3) 1,5 кДж/(кгК)

4) 3 кДж/(кгК)

А 6

Для нагревания кирпича массой 2 кг от 20 ºC до 85 º C затрачено такое же количество теплоты, как для нагревания той же массы воды на     13 º C. Удельная теплоёмкость воды 4200 . Удельная теплоёмкость кирпича равна

1) 840

2) 21000

3) 2100

4) 1680

А 7

Если к твердым веществам с одинаковой массой и одинаковой начальной температурой подвести одинаковое количество теплоты и они останутся твердыми, то температура вещества с большей удельной теплоёмкостью

1) будет такой же, как у второго тела

2) будет выше, чем у второго тела

3) будет ниже, чем у второго тела

4) может быть выше и ниже, чем у второго, в зависимости от времени теплопередачи

А 8

При передаче твердому телу массой  количества теплоты   температура тела повысилась на . Какое из приведенных ниже выражений определяет теплоёмкость вещества этого тела?

1)                  2)                  3)                     4)

А 9

При охлаждении твердого тела массой  температура тела понизилась на . Какое из приведенных ниже выражений определяет  теплоёмкость вещества этого тела, если при этом охлаждении тело передало окружающим телам количество теплоты  ?

1)                   2) 

3)             4)

3

4. Теплообмен без агрегатных переходов

А 1

Воздух в комнате состоит из смеси газов: водорода, кислорода, азота, водяного пара, углекислого газа и др. При тепловом равновесии у этих газов обязательно одинаковы

1) температуры

2) парциальные давления

3) концентрации молекул

4) плотности

А 2

Тело А находится в тепловом равновесии с телом С, а тело В не находится в тепловом равновесии с телом С. Найдите верное утверждение

1) температуры тел А и В одинаковы

2) температуры тел А, С и В одинаковы

3) тела А и В находятся в тепловом равновесии

4) температуры тел А и В не одинаковы

А 3

Теплопередача всегда происходит от тела с

1)

большим запасом количества теплоты к телу с меньшим запасом количества теплоты

2)

большей теплоемкостью к телу с меньшей теплоёмкостью

3)

большей температурой к телу с меньшей температурой

4)

большей теплопроводностью к телу с меньшей теплопроводностью

А 4

Температура тела А равна 300 К, температура тела Б равна 100 оС. Температура какого из тел повысится  при тепловом контакте тел?

1) тела А

2) тела Б

3) температуры тел А и Б не изменятся

4) температуры тел А и Б могут только понижаться

А 5

Температура тела А равна 373,15 К, а температура тела Б равна 100 оС. При тепловом контакте этих тел

1) температура тела А повышается

2) температура тела Б повышается

3) температура тел А и Б не изменятся

4) температура тел А и Б  понижается

А 6

Тепловой контакт двух сосудов с газами при разных температурах осуществляется способом А и способом Б. Если теплообмен возможен только между сосудами, то средняя кинетическая энергия молекул в верхнем сосуде увеличивается

1) только в случае А

2) только в случае Б

3) в обоих случаях

4) ни в одном из случает

4

А 7

При измерении температуры жидкости рекомендуется подождать некоторое время, прежде чем записывать показания термометра. Это объясняется тем, что

1) жидкость испаряется

2) жидкость плохо сжимается

3) жидкость обладает текучестью

4) должно установиться тепловое равновесие между термометром и жидкостью

Два вещества

В 1

Для определения удельной теплоёмкости вещества тело массой 450 г, нагретое до температуры 100 оС, опустили в калориметр, содержащий 200 г воды. Начальная температура калориметра с водой 23 оС. После установления теплового равновесия температура тела и воды стала равна 30 оС. Определите удельную теплоёмкость вещества исследуемого тела. Удельная теплоёмкость воды равна                    4200. Теплоёмкостью калориметра пренебречь.            

В 2

Для определения удельной теплоёмкости вещества тело массой 400 г, нагретое до температуры 100 оС, опустили в калориметр, содержащий 200 г воды. Начальная температура калориметра с водой 30 оС. После установления теплового равновесия температура тела и воды стала равна 37 оС. Определите удельную теплоёмкость вещества исследуемого тела. Удельная теплоёмкость воды равна                    4200. Теплоёмкостью калориметра пренебречь               

В 3

Тело массой 300 г, нагретое до температуры 100 оС, опустили в стакан калориметра, содержащего  200 г воды. Начальная температура калориметра с водой 23 оС. После установления теплового равновесия температура тела и воды стала равна 30 оС. Определите удельную теплоёмкость вещества исследуемого тела. Удельная теплоёмкость воды равна 4200. Теплоёмкостью стакана калориметра пренебречь.        

В 4

При измерении удельной теплоёмкости алюминия образец массой    100 г был нагрет в кипящей воде, а затем опущен в 100 г воды при температуре 20 оС. В результате теплообмена установилась температура 35 оС. Чему равна удельная теплоёмкость алюминия? Удельная теплоёмкость воды равна 4200. Тепловыми потерями пренебречь.  

Три вещества

В 5

Для определения удельной теплоёмкости вещества тело массой 500 г, нагретое до температуры 100 оС, опустили в железный стакан калориметра, содержащий 200 г воды. Начальная температура калориметра с водой 30 оС. После установления теплового равновесия температура тела, воды  и калориметра стала равна 37 оС. Определите удельную теплоёмкость вещества исследуемого тела. Масса  калориметра  равна 100 г, удельная теплоёмкость железа равна          640 , удельная теплоёмкость воды равна 4180            

В 6

Для определения удельной теплоёмкости вещества тело массой 400 г, нагретое до температуры 100 оС, опустили в железный стакан калориметра, содержащий 200 г воды. Начальная температура калориметра с водой 30 оС. После установления теплового равновесия температура тела, воды  и калориметра стала равна 37 оС. Определите удельную теплоёмкость вещества исследуемого тела. Масса  калориметра  равна 100 г, удельная теплоёмкость железа равна          640 , удельная теплоёмкость воды равна 4180            

В 7

Для определения удельной теплоёмкости вещества тело массой 1 кг, нагретое до температуры 100 оС, опустили в железный стакан калориметра, содержащий 600 г воды. Начальная температура калориметра с водой 30 оС. После установления теплового равновесия температура тела, воды  и калориметра стала равна 37 оС. Определите удельную теплоёмкость вещества исследуемого тела. Масса  калориметра  равна 300 г, удельная теплоёмкость железа равна          640, удельная теплоёмкость воды равна  4180            

В 8

Для определения удельной теплоёмкости вещества тело из этого вещества массой 400 г, нагретое до температуры 100 оС, опустили в железный стакан калориметра, содержащий 200 г воды. Начальная температура калориметра с водой 30 оС. После установления теплового равновесия температура тела,  воды и калориметра оказалась  равна 40 оС. Определите удельную теплоёмкость вещества исследуемого тела. Масса калориметра 100 г, удельная теплоёмкость железа равна 640,воды  4180.

5. Плавление и кристаллизация

А 1

При плавлении кристаллическое твердое тело становится жидкостью. Это проявляется в том, что

1) уменьшается внутренняя энергия тела

2) разрушается кристаллическая решетка

3) повышается температура твердого тела

4) изменяется химический состав вещества

А 2

Плавление вещества происходит потому, что

1) частицы с любыми скоростями покидают твёрдое тело

2) частицы уменьшаются в размерах

3) уменьшается потенциальная энергия частиц твёрдого тела

4) разрушается кристаллическая решетка

А 3

В процессе плавления кристаллического тела происходит

1) уменьшение размеров частиц

2) изменение  химического состава

3) разрушение кристаллической решетки

4) уменьшение кинетической энергии частиц

А 4

Как изменяется внутренняя энергия кристаллического вещества в процессе его плавления

1) увеличивается

2) уменьшается

3) не изменяется

4) для одних кристаллических веществ увеличивается, для других - уменьшается

А 5

Температура кристаллического тела при плавлении не изменяется. Внутренняя энергия вещества  при плавлении

1) увеличивается

2) не изменяется

3) уменьшается

4) может увеличиваться или уменьшаться в зависимости от кристаллической структуры тела

А 6

Медь плавится при постоянной температуре 1085 оС. Поглощается или выделяется энергия в этом процессе?

1) поглощается

2) выделяется

3) не поглощается и не выделяется

4) может поглощаться, может выделяться

А 7

Лед при температуре 0 оС внесли в теплое помещение. Температура льда до того, как он растает,

1) не изменится, так как вся энергия, получаемая льдом в это время, расходуется на разрушение кристаллической решетки

2) не изменится, так как при плавлении лёд получает тепло от окружающей среды, а затем отдаёт обратно

3) повысится, так как лёд получает тепло от окружающей среды, значит, его внутренняя энергия растёт, и температура льда повышается

4) понизится, так как при плавлении лед отдаёт окружающей среде некоторое количество теплоты

5

А 8

Весной при таянии льда в водоёме температура окружающего воздуха:

1)

уменьшается

2)

увеличивается

3)

не изменяется

4)

может увеличиваться или уменьшаться

А 9

При замерзании воды энергия

1) выделяется

2) поглощается

3) не выделяется и не поглощается

4) в одних условиях может выделяться, а в других - поглощаться

А 10

Вещество массой  находится в твёрдом состоянии. К нему при постоянной температуре  подводят количество теплоты , и оно переходит в жидкое состояние. Удельную теплоту плавления можно рассчитать по формуле:

1)                  2)                3)             4)

А 11

В таблице приведена зависимость температуры плавления ряда веществ от их молярной массы

Вещество

М, кг/моль

Вещество

М, кг/моль

Литий

7

180

Кремний

28

1410

Бериллий

9

900

Сера

32

113

Бор

11

2300

Хлор

35,5

-101

Фтор

19

-220

Калий

39,1

64

Неон

20

-248

Кальций

40

839

Натрий

23

98

Скандий

44

1541

Алюминий

27

660

Титан

48

1660

На основании этих данных, можно заключить, что для данной группы веществ температура плавления с ростом молярной массы вещества

1) монотонно увеличивается

2) монотонно уменьшается

3) периодически растет, а затем спадает

4) изменяется случайным образом с ростом молярной массы

А 12

- плотность вещества в жидком состоянии, -  после кристаллизации. Какое соотношение плотностей справедливо

1)    2)      3)     4) зависит от вещества

6

6. Кипение и конденсация. Температура кипения. Испарение

А 1

Кипение жидкости происходит при постоянной температуре. Для кипения  необходим постоянный приток энергии. Подводимая к жидкости энергия расходуется на

1) увеличение средней кинетической энергии молекул жидкости

2) уменьшение средней кинетической энергии молекул жидкости

3) установление динамического равновесия между жидкостью и паром

4) совершение работы выхода молекул с поверхности жидкости

А 2

Вода кипит при определенной постоянной температуре. Температуру кипения воды можно понизить, если

1) добавить в воду поваренную соль

2) уменьшить давление воздуха и водяных паров в сосуде

3) размешивать воду

4) отлить часть воды из сосуда

А 3

Температура кипения воды зависит от

  1.  мощности нагревателя
  2.  вещества сосуда, в котором нагревается вода
  3.  атмосферного давления
  4.  начальной температуры воды

А 4

При повышении давления  над жидкостью температура её кипения:

1) повышается

2) понижается

3) не изменяется

4) для одних жидкостей повышается, а для других понижается

А 5

Какова температура кипения воды при нормальном атмосферном давлении по абсолютной шкале температур?

1) 100 К                2) 173 К                      3) 273 К                   4) 373 К

А 6

В электрочайнике неисправный нагреватель заменили на нагреватель вдвое меньшей мощности. Температура воды при этом

1) уменьшилась в 2 раза

2) уменьшилась более, чем в 2 раза

3) уменьшилась меньше, чем в 2 раза

4) практически не изменилась

А 7

При конденсации парообразного вещества выделяется энергия. Это происходит за счёт уменьшения

1) потенциальной  энергии взаимодействия между молекулами вещества

2) кинетической энергии молекул вещества

3) энергии взаимодействия между ядрами атомов близких молекул

4) энергии взаимодействия молекул вещества с Землёй

А 8

В сосуде находится жидкость и её пар. В процессе конденсации пара выделяется некоторое количество теплоты. При этом внутренняя энергия вещества

1) увеличивается

2) уменьшается

3) не изменяется

4) превращается в механическую энергию

А 9

С поверхности воды происходит испарение при отсутствии теплообмена с внешними телами. Как в результате этого процесса изменяется внутренняя энергия испарившейся и оставшейся воды?

1) внутренняя энергия испарившейся воды увеличивается, внутренняя энергия оставшейся воды уменьшается

2) внутренняя энергия испарившейся воды уменьшается, внутренняя энергия оставшейся воды увеличивается

3) внутренняя энергия испарившейся воды увеличивается, внутренняя энергия оставшейся воды  не изменяется

4) внутренняя энергия испарившейся воды уменьшается, внутренняя энергия оставшейся воды не изменяется

А 10

Если при атмосферном давлении 100 кПа конденсируется 200 г паров некоторого вещества при 100 оС, то в окружающую среду передаётся количество теплоты, равное 460 кДж. Удельная теплота парообразования этого вещества приблизительно равна

1)

2)

3)

4)

А 11

При отводе от вещества в газообразном состоянии количества теплоты  при постоянной температуре  происходит превращение вещества массой  из газообразного  состояния в жидкое. Какое выражение определяет удельную теплоту конденсации этого вещества?

1)                  2)                  3)           4)

7. Графики тепловых процессов

А 1

На графике (см. рисунок) показан график зависимости температуры Т вещества от времени t. В начальный момент времени вещество находилось в кристаллическом состоянии. Какая из точек соответствует началу  процесса плавления вещества?

1)

5

2)

2

3)

3

4)

6

А 2

На графике (см. рисунок) показан график зависимости температуры Т вещества от времени t. В начальный момент времени вещество находилось в кристаллическом состоянии. Какая из точек соответствует окончанию  процесса плавления вещества?

1)

5

2)

2

3)

3

4)

6

А  3

На  рисунке изображен график плавления и кристаллизации нафталина. Какая из точек соответствует началу  отверде-вания  вещества?

1)

2

2)

4

3)

5

4)

6

А 4

На графике (см. рисунок) представлено изменение температуры Т вещества с течением времени t. В начальный момент времени вещество находилось в кристаллическом состоянии. Какая из точек соответствует окончанию процесса отвердевания?

1)

5

2)

6

3)

3

4)

7

А 5

На  рисунке показан график зависимости  температуры  эфира от времени t его нагревания и охлаждения. Какой участок  графика соответствует  процессу кипения эфира?

1)

1-2

2)

2-3

3)

1-2-3

4)

3-4

7

А 6

На рисунке представлен график зависимости абсолютной температуры T воды массой m от времени t при осуществлении теплопередачи с постоянной мощностью P. В момент времени = 0 вещество  находилось в твердом состоянии. В течение какого интервала времени происходило нагревание льда,  и в каком интервале происходило его плавление?

1)

и

2)

и

3)

и

4)

и

А 7

На рисунке представлен график зависимости абсолютной температуры T воды массой m от времени t при осуществлении теплопередачи с постоянной мощностью P. В момент времени t = 0 вещество  находилось в твердом состоянии. В течение какого интервала времени происходило плавление льда,  и в каком интервале происходило нагревание водяного пара?

1)

и

2)

и

3)

и

4)

и

8

А 8

На рисунке представлен график зависимости абсолютной температуры T воды массой m от времени t при осуществлении теплопередачи с постоянной мощностью P. В момент времени t = 0 вода  находилась в твердом состоянии. В течение какого интервала времени происходило нагревание льда,  и в каком интервале происходило нагревание водяного пара?

1)

и

2)

и

3)

и

4)

и

А 9

На рисунке представлен график зависимости абсолютной температуры T воды массой m от времени t при осуществлении теплоотвода с постоянной мощностью P. В момент времени t = 0 вещество  находилось в газообразном состоянии. В течение каких интервалов времени происходило охлаждение жидкого и твердого вещества?

1)

и

2)

и

3)

и

4)

и

А 10

На рисунке представлен график зависимости абсолютной температуры T воды массой m от времени t при осуществлении теплоотвода с постоянной мощностью P. В момент времени t = 0 вода находилась в газообразном состоянии. Какое из приведенных ниже выражений определяет удельную теплоемкость льда по результатам этого опыта?

1)

2)

3)

4)

А 11

На рисунке представлен график зависимости абсолютной температуры T воды массой m от времени t при осуществлении теплоотвода с постоянной мощностью P. В момент времени = 0 вещество находилось в газообразном состоянии. Какое из приведенных ниже выражений определяет удельную теплоемкость жидкости по результатам этого опыта?

1)

2)

3)

4)

А 12

На рисунке представлен график зависимости абсолютной температуры T воды массой m от времени t при осуществлении теплопередачи с постоянной мощностью P. В момент времени t = 0 вода  находилась в твердом состоянии. Какое  из приведенных ниже выражений определяет удельную теплоёмкость водяного пара по результатам  этого опыта?

1)

2)

3)

4)

Плавление и отвердевание

А13

На рисунке показан график зависимости температуры кристаллического вещества от   времени его нагревания. Какова температура  плавления  вещества?

1) 80 оС                   2) 60 оС                   3) 50 оС              4) 45 оС

А14

На рисунке  показан график изменения температуры вещества по мере поглощения  теплоты. Масса тела 0,15 кг. Первоначально вещество было в твердом состоянии. Какова удельная теплота плавления вещества?

1) Дж/кг

3) Дж/К

2)  Дж/К

4) Дж/К

9

А15

В котелок насыпали снег и поставили на электрическую плитку. Плитка передаёт котелку в среднем 500 Дж энергии в минуту. Диаграмма изменения температуры снега с течением времени показана на рисунке. Какое количество теплоты потребовалось для плавления снега?

1) 2500 Дж         2) 5000 Дж 

3) 7500 Дж             4) 12500 Дж

А16

В  печь поместили некоторое количество алюминия. Диаграмма изменения температуры алюминия с течением времени  показана на рисунке. Печь при постоянном нагреве передаёт алюминию       2 кДж энергии в минуту. Какое количество теплоты потребовало  плавление  алюминия?

1) 5 кДж          2) 15  кДж

3) 20 кДж             4) 30 кДж

А17

В керамическую чашечку (тигель) опустили электрический термометр и насыпали опилки олова. После этого тигель поместили в печь. Диаграмма изменения температуры олова с течением времени показана на рисунке. Печь при постоянном нагреве передавала олову 500 Дж энергии в минуту. Какое количество теплоты потребо-валось для плавления олова?

1) 2500 Дж            2) 2000 Дж

3) 1500 Дж            4) 500 Дж

А18

Для плавления куска олова требуется 5,4 кДж энергии. Этот кусок положили в печь. Зависимость  температуры олова от времени нагревания представлена  на рисунке. С какой скоростью печь передавала тепло олову?

1) 900 Дж/мин

3) 225 Дж/мин

2) 300 Дж/мин

4) 180 Дж /мин

10

А19

На каком из графиков правильно изображена  зависимость температуры  от   времени  в сосуде, который наполнен льдом и поставлен на горелку? Удельная теплоёмкость воды больше удельной теплоёмкости льда. Мощность горелки считать постоянной.

1)

2)

3)

4)

С 1

В калориметре нагревается 200 г льда. На рисунке представлен график зависимости температуры вещества от времени. Пренебрегая теплоемкостью калориметра и тепловыми потерями, определите подводимую к нему  мощность при рассмотрении процессов нагревания льда или воды.        Удельная теплоёмкость воды 4200 , а льда                     2100  

С 2

В калориметре нагревается 200 г вещества. На рисунке представлен график зависимости температуры вещества в калориметре от времени. Пренебрегая теплоемкостью калориметра и тепловыми потерями и предполагая, что подводимая к сосуду мощность постоянна, определите удельную теплоемкость твердого вещества, если удельная теплоемкость жидкости cж = 2,8 кДж/кгК.  

С 3

В калориметре нагревается лед массой 200 г. На рисунке представлен график зависимости температуры льда от времени. Пренебрегая теплоемкостью калориметра и тепловыми потерями, определите удельную теплоту плавления льда из рассмотрения процессов нагревания льда и воды. Удельная теплоёмкость воды 4200 , а льда     2100  

С 4

На рисунке представлен график зависимости температуры вещества в калориметре от времени. Теплоемкостью калориметра и тепловыми потерями можно пренебречь и считать, что подводимая к сосуду мощность постоянна. Определите удельную теплоемкость жидкости. Удельная теплота плавления вещества равна  100 кДж/кг. В первый момент времени вещество находилось в твердом состоянии.

Кипение и конденсация

А 20

На рисунке приведены графики изменения со временем температуры четырёх веществ. В начале нагревания все эти вещества находились в жидком состоянии. Какое из веществ имеет наибольшую температуру кипения?

1) 1                             2) 2                          

3)  3                              4) 4

А 21

На графике показаны кривые нагревания двух жидкостей одинаковой массы при постоянной мощности подводимого тепла. Отношение температур кипения первого вещества к температуре кипения второго  вещества?

1) 1/3                     2) 1/2                     3) 2                             4) 3

А 22

На графике показаны кривые нагревания двух жидкостей одинаковой массы при постоянной мощности подводимого тепла. Отношение удельной теплоты парообразования первого вещества к удельной теплоте парообразования второго   равно

1) 1/3                     2) 1/2                     3) 2                         4) 3

Количество теплоты и изменение температуры

А 23

На рисунке  приведен график зависимости температуры твердого тела от полученного им количества теплоты. Масса тела 8 кг. Какова удельная тепло-ёмкость вещества этого тела?

1) 2,5 Дж/(кгК)

2) 625 Дж/(кгК)

3) 2500 Дж/(кгК)

4) 10000 Дж/(кгК)

А 24

На рисунке  приведен график зависимости температуры твердого тела от полученного им количества теплоты. Масса тела 4 кг. Какова удельная тепло-ёмкость вещества этого тела?

1) 0,002 Дж/(кгК)

2) 0,5 Дж/(кгК)

3) 500 Дж/(кгК)

4) 40000 Дж/(кгК)

А 25

На рисунке показаны графики зависимости температур I и II тел одинаковой массы от подведенного к ним количества теплоты (рис.). Известно, что тело I изготовлено из цинка. Пользуясь приведенной в Инструкции таблицей, определите, какое из веществ могло быть использовано для изготовления тела II.

1) натрий           2) железо

3) бериллий       4) свинец

11

А 26

На рисунке показаны графики зависимости температур I и II тел одинаковой массы от подведенного к ним количества теплоты (рис.). Известно, что тело II изготовлено из цинка. Пользуясь приведенной в Инструкции таблицей, опреде-лите, какое из веществ могло быть использовано для изгото-вления тела I.

1) свинец

2) железо

3) бериллий       

4) натрий

А 27

На рисунке показаны графики зависимости температур I и II тел одинаковой массы от подведенного к ним количества теплоты (рис.). Известно, что тело II изготовлено из никеля. Пользуясь приведенной в Инструкции таблицей, опреде-лите, какое из веществ могло быть использовано для изгото-вления тела I.

1) только алюминий

2) только олово

3) только серебро      

4) как серебро, так и олово

А 28

На рисунке показаны графики зависимости температур I и II тел одинаковой массы от подведенного к ним количества теплоты (рис.). Известно, что тело I изготовлено из меди. Пользуясь приведенной в Инструкции таблицей, опреде-лите, какое из веществ могло быть использовано для изгото-вления тела II.

1) свинец

2) цинк

3) натрий

4) серебро

12

А 29

На рисунке показаны графики зависимости температур I и II тел одинаковой массы от подведенного к ним количества теплоты (рис.). Известно, что тело II изготовлено из меди. Пользуясь приведенной в Инструкции таблицей, опреде-лите, какое из веществ могло быть использовано для изгото-вления тела I.

1) свинец                 2) цинк

3) натрий                  4) серебро

А 30

Экспериментально исследовалась зависимость времени закипания некоторого количества вещества воды от мощности кипятильника. По результатам  измерений построен график, приведенный на рисунке. Какой вывод можно сделать по результатам эксперимента?

  1.  время нагревания прямо пропорционально мощности нагревателя.
  2.  с ростом мощности нагревателя вода нагревается быстрее
  3.  мощность  нагревателя с течением времени уменьшается
  4.  теплоёмкость воды равна 4200 Дж/(кгК)

А 31

На графике показана зависимость температуры  воды  от времени. Если изменение температуры  воды происходит только за счет теплопередачи, то такой ход графика возможен в случае:

1) на участке ОМ вода находится в контакте с более горячим

телом, а на участке     и   МN - с более холодным 2) на участке ОМ вода находится в контакте с более холодным телом, а на участке MN - с более горячим

3) на участках ОМ и МN вода находится в контакте с более горячим телом

4) на участках ОМ и МN вода находится в контакте с более холодным телом

А 32

На графике показана зависимость температуры жидкости от времени. Если изменение температуры жидкости происходит только за счет теплопередачи, то такой ход графика возможен в случае:

1) на участке ОМ жидкость находится  в контакте    с    более

горячим телом, а на   участке МN - с более холодным     

2) на участке ОМ жидкость находится в контакте с более холодным телом, а на участке МN - с более горячим

3) на участках ОМ и МN жидкость находится в контакте с более горячим телом

4) на участках ОМ и МN жидкость находится в контакте с более холодным телом

А 33

На графике показана зависимость температуры воды в чайнике от времени. Такой ход графика возможен, если

1) первые 20 минут чайник стоял на горячей плите, а вторые 20 минут – на столе

2) первые 20 минут чайник стоял на столе, а вторые 20 минут - на горячей плите

3) все 40 минут чайник стоял на столе

4) все 40 минут чайник стоял на горячей плите

А 34

На графике показана зависимость температуры воды в кружке от времени. Такой ход графика возможен, если кружка с водой

1) первые 20 минут стояла в морозильной   камере   при температуре    - 15 °С, а вторые 20 минут - на столе при температуре 20 °С

2) первые 20 минут стояла на столе при температуре 20 °С, а вторые   20 минут - в морозильной камере при температуре  - 15 °С

3) все 40 минут стояла на столе при температуре 20 °С

4) все 40 минут стояла в морозильной камере при температуре        -15 °С

А 35

На графике показана зависимость температуры воды в чайнике от времени. Такой ход графика возможен, если

  1.  первые 20 минут чайник стоял на горячей плите, а вторые             20 минут - на столе
  2.  первые 20 минут чайник стоял на столе, а вторые 20 минут - на горячей  плите
  3.   все 40 минут чайник стоял на столе
  4.   все 40 минут чайник стоял на горячей плите

 

А36

Горячая жидкость медленно охлаждалась  в стакане. В таблице приведены результаты  измерений ее температуры  с течением времени.

Время, мин

0

2

4

6

8

10

12

14

Температура, оС

95

88

81

80

80

80

77

72

В стакане через 7 мин после начала измерений находилось вещество

  1.  только в жидком состоянии
  2.  только в твердом состоянии
  3.  и в жидком, и в твердом состояниях
  4.   и в жидком, и в газообразном состояниях

Формулы и физические константы тепловых процессов

В 1

Установите соответствие между названием физической величины  и формулой, по которой ее можно определить.

НАЗВАНИЕ

ФОРМУЛА

А) Количество теплоты, необходимое для нагревания тела

Б) Удельная теплота плавления кристаллического вещества

В) Количество теплоты, выделяемое при сгорании топлива

1)

2)

3)

4)

5)

А

Б

В

13

В 2

Установите соответствие между названием физической величины  и формулой, по которой ее можно определить.

НАЗВАНИЕ

ФОРМУЛА

А) Количество теплоты, необходимое для плавления  твердого тела

Б) Удельная теплоёмкость  вещества

В) Количество теплоты,  выделяемое при конденсации пара

1)

2)

3)

4)

5)

А

Б

В

В 3

Установите соответствие между названием физической величины  и формулой, по которой ее можно определить.

НАЗВАНИЕ

ФОРМУЛА

А) Количество теплоты, необходимое для кипения жидкости

Б) Удельная теплота сгорания топлива

В) Количество теплоты, выделяемое при охлаждении вещества

1)

2)

3)

4)

5)

А

Б

В

14

В 4

Установите соответствие между названием физической величины  и формулой, по которой ее можно определить.

НАЗВАНИЕ

ФОРМУЛА

А) Количество теплоты, выделяемое при  кристаллизации вещества

Б) Удельная теплота  парообразования

В) Количество теплоты, выделяемое при сгорании топлива

1)

2)

3)

4)

5)

А

Б

В

8. Кристаллические и аморфные тела

А 1

Какие из приведенных свойств,  принадлежат всем твердым телам?

А. имеют определенный объём

Б. имеют кристаллическую решетку

В. принимают форму сосуда

С. легко сжимаются

1) А

2) А,Б

3) В    

4) Г

А 2

Что характерно  для  кристаллических тел?

А.  обладают анизотропией

Б. сохраняют форму

В. сохраняют объём

Г. переходят в жидкое состояние только при определенной температуре – температуре плавления

1) А  

2) А, Б          

3) А, Б, В                

4) А, Б, В, Г

А 3

Что характерно только для  аморфных тел?

А. не имеют кристаллическую решетку

Б.  постепенно переходят из твердого состояния в жидкое.

В. обладают анизотропией

Г. сохраняют форму и объём

1) А

2) А, Б           

3) А, Б, Г               

4) А, Б, В, Г               

А 4

При нагревании двух твердых тел из кристаллического (1) и аморфного (2) вещества переход в жидкое состояние

1) происходит резко при достижении определенной температуры и для 1,  и  для второго тела

2) происходит резко при достижении определенной температуры только для 1 тела

3) происходит резко при достижении определенной температуры только для 2 тела

4) происходит постепенно для обоих тел, сопровождаясь повышением температуры смеси жидкого и твердого вещества

А 5

В таблице указаны результаты измерения температуры твердого кристаллического вещества с температурой плавления 220 оС спустя время  после начала равномерного нагревания его на электроплитке. Ошибка в измерениях температуры 1 оС

, мин

5

10

15

20

48

100

145

190

Можно утверждать, что в сосуде после начала нагревания при неизменных условиях находятся

1) через 15 мин – твердое тело, через 30 мин – твердое тело

2) через 15 мин – жидкость, через 30 мин – жидкость

3) через 15 мин – жидкость, через 30 мин – твердое тело

4) через 15 мин – твердое тело, через 30 мин – жидкость

А 6

Какое свойство отличает кристалл от аморфного тела

1) анизотропность

2) прозрачность

3) твердость

4) прочность

А 7

Имеется кубик кристалла соли и кубик такого же размера из аморфного стекла. Из кубика соли вырезают столбики одинакового размера. Вдоль ребра и вдоль диагонали  столбиков  проводят испытания на разрыв. Получают  отношение нагрузок, при которых происходит разрушение столбиков. Затем проводят такие же измерения для столбиков, вырезанных вдоль этих направлений из стеклянного кубика. Полученное отношение нагрузок

1) равно 1 для соли и для стекла

2) не равно 1 для соли и для стекла

3) равно 1 для соли и не равно 1 для стекла

4) не равно 1 для соли и  равно 1 для стекла

9. КПД нагревателя

В 1

В электрический кофейник налили воду объёмом 0,16 л при температуре 30 оС и включили нагреватель. Через какое время после включения выкипит вся вода, если мощность нагревателя 1 кВт, КПД нагревателя 0,8? Удельная теплоёмкость воды 4200 . Удельная теплота парообразования воды 2256 кДж/кг. 

В 2

В электрический кофейник налили воду объёмом 0,8 л при температуре 30 оС и включили нагреватель. Через какое время после включения выкипит вся вода, если мощность нагревателя 1 кВт, КПД нагревателя 0,8? Удельная теплоёмкость воды 4200 . Удельная теплота парообразования воды 2256 кДж/кг. 

В 3

В электрический кофейник налили воду объёмом 0,45 л при температуре 30 оС и включили нагреватель. Через какое время после включения выкипит вся вода, если мощность нагревателя 1 кВт, КПД нагревателя 0,9? Удельная теплоёмкость воды 4200 . Удельная теплота парообразования воды 2256 кДж/кг. 

В 4

В электрический кофейник налили воду объёмом 0,75 л  и включили нагреватель. Через 20 мин после включения вся вода выкипела.  Мощность нагревателя 2 кВт, КПД нагревателя 0,8. Удельная теплоёмкость воды 4200 . Удельная теплота парообразования воды 2256 кДж/кг. Какова начальная температура воды?

10. Взаимные превращения механической и внутренней энергии

В 1

При какой скорости пули из свинца она полностью расплавится при ударе о стенку, если 80 % ее энергии будет затрачено на нагревание пули? Начальная температура пули 27 оС, температура плавления свинца 327 оС, удельная теплоёмкость  130 , удельная теплота плавления 25 кДж/кг.

11. Теплообмен с агрегатными переходами

В 1

В теплоизолированный сосуд с большим количеством льда при температуре 0 оС заливают 1 кг воды с температурой 44 оС. Какая масса льда расплавится при установлении теплового равновесия в сосуде? Ответ выразите в граммах. Удельная теплоёмкость воды равна 4200 , удельная теплота плавления льда 330 кДж/кг. 

В 2

В стакан калориметра, содержащий 177 г воды, опустили кусок льда, имевший температуру  0 оС. Начальная температура калориметра с водой равна 45 оС. После того, как весь лёд растаял, температура воды и калориметра стала равна 5 оС. Определите массу льда. Теплоёмкостью калориметра пренебречь. Удельная теплоёмкость воды равна 4200 , удельная теплота плавления льда        330 кДж/кг. 

15

В 3

Воду массой 100 г при температуре 12 °С поместили в калориметр, где находился лед при температуре - 5 °С. После установления теплового равновесия, температура льда повысилась до 0 °С, но масса льда не изменилась. Пренебрегая потерями тепла, оцените, какова была начальная масса льда в калориметре. Удельная теплоёмкость льда равна              2100 , удельная теплоёмкость воды равна 4200  .

В 4

В сосуд с водой  бросают кусочки тающего льда,  при непрерывном помешивании, вначале кусочки льда тают, но в некоторый момент лёд перестаёт таять.  Первоначальная масса воды в сосуде  330 г. В конце процесса масса воды увеличилась. На сколько увеличилась масса воды, если её первоначальная температура  20 оС. Удельная теплоёмкость воды 4200, удельная теплота плавления льда  330 кДж/кг. 

В 5

В сосуд с водой  бросают кусочки тающего льда,  при непрерывном помешивании. Кусочки льда тают, но в некоторый момент лёд перестаёт таять. К концу процесса  масса воды увеличилась  на 84 г.  Найдите начальную массу воды, если  её первоначальная температура  20 оС. Удельная теплоёмкость воды 4200, удельная теплота плавления льда  330 кДж/кг. 

В 6

В калориметр с водой  бросают кусочки тающего льда. В некоторый момент  кусочки льда  перестают таять.  Первоначальная масса воды в сосуде  330 г, а в конце процесса масса воды увеличивается на 84 г. Какой была начальная температура  воды в калориметре?  Удельная теплоёмкость воды 4200, удельная теплота плавления льда  330 кДж/кг. 

В 7

Для определения удельной теплоты плавления льда в сосуд с водой  бросают кусочки тающего льда при непрерывном помешивании. Первоначально в сосуде находилось 300 г воды при температуре 20 оС. К моменту времени, когда лёд перестал таять, масса воды увеличилась на 84 г. Определите по этим данным удельную теплоту плавления льда. Удельная теплоёмкость воды 4200

В 8

Воду массой 500 г при температуре 95 оС налили в теплоизолированный сосуд, где находился твердый нафталин при температуре       80 оС. После установления теплового равновесия, температура воды оказалась равна 80 °С, при этом весь нафталин перешел в жидкое состояние. Пренебрегая потерями тепла, оцените, чему равна масса нафталина в сосуде. Удельная теплоёмкость воды равна 4200, удельная теплота плавления нафталина 150 кДж/кг. Температура плавления нафталина 80 °С.          

16

В 9

Твердый нафталин находится в теплоизолированном сосуде при температуре 80 °С. В сосуд наливают расплавленный нафталин массой 600 г, начальная температура которого равна 100°С. С некоторого момента кусочки нафталина в сосуде перестают плавиться, а масса жидкого нафталина в сосуде в этот момент равна 700 г. По результатам этого эксперимента определите удельную теплоемкость жидкого нафталина. Ответ выразите в Дж/(кгК). Удельная теплота плавления нафталина 150 кДж/кг. Температура плавления нафталина 80 °С.

В 10

Жидкий парафин массой 200 г при температуре 54 °С налили в калориметр, где находилась вода при температуре 40 °С. После установления теплового равновесия, температура воды возросла до 54 °С, при этом весь парафин затвердел. Пренебрегая потерями тепла, оцените, чему равна масса воды в калориметре. Ответ округлите до сотых. Удельная теплоёмкость воды равна 4200, удельная теплота плавления парафина 150 кДж/кг. Температура плавления парафина 54 °С.

В 11

Для охлаждения лимонада массой 200 г в него бросают кубики льда при  0 оС. Масса каждого кубика 8 г. Первоначальная температура лимонада 30 оС. Сколько целых кубиков надо бросить в лимонад, чтобы установилась температура 15 оС? Тепловыми потерями пренебречь. Удельная теплоёмкость лимонада такая же, как у воды. Удельная теплоёмкость воды 4200, удельная теплота плавления льда  330 кДж/кг.

В 12

Для охлаждения лимонада массой 200 г в него бросают кубики льда при  0 оС. Масса каждого кубика 8 г. Какова первоначальная температура лимонада, если  установилась температура 15 оС после того, как в него бросили 4 кубика? Тепловыми потерями пренебречь. Удельная теплоёмкость лимонада такая же, как у воды. Удельная теплоёмкость воды 4200, удельная теплота плавления льда  330 кДж/кг. 

В 13

Воду массой 300 г при температуре 95 °С налили в теплоизолированный сосуд, где находился твердый нафталин при температуре       80 °С. После установления теплового равновесия, температура воды оказалась равна 80 °С, при этом весь твердый нафталин перешел в жидкое состояние. Пренебрегая потерями тепла, оцените, чему равна масса нафталина в сосуде. Ответ выразите в граммах. Удельная теплоёмкость воды равна 4200, удельная теплота плавления нафталина 150 кДж/кг. Температура плавления нафталина 80 °С.         

В 14

В медный стакан калориметра массой 200 г, содержащий воду массой 150 г, опустили кусок льда, имеющий температуру 0 оС. Начальная температура калориметра с водой равна 25 оС. После того, как весь лёд растаял, температура воды и калориметра стала равна 5 оС. Определите массу льда. Ответ округлите до сотых. Удельная теплоёмкость меди равна  390 , удельная теплоёмкость воды равна 4200  , удельная теплота плавления льда       333 кДж/кг.           

В 15

В медный стакан калориметра массой 100 г, содержащий   75 г воды, опустили кусок льда, имеющий температуру 0 оС. Начальная температура калориметра с водой равна 45 оС. После того, как весь лёд растаял, температура воды и калориметра стала равна 5 оС. Определите массу льда. Ответ округлите до сотых. Удельная теплоёмкость меди равна  390 , удельная теплоёмкость воды равна 4200  , удельная теплота плавления льда      333 кДж/кг.         

В 16

В медный стакан калориметра массой 200 г, содержащий воду массой 150 г, опустили кусок льда, имеющий температуру 0 оС. Начальная температура калориметра с водой равна 45 оС. После того, как весь лёд растаял, температура воды и калориметра стала равна 5 оС. Определите массу льда. Ответ округлите до сотых. Удельная теплоёмкость меди равна    390 , удельная теплоёмкость воды равна 4200  , удельная теплота плавления льда      333 кДж/кг.         

С 1

В медный стакан калориметра массой 200 г, содержащий воду массой 200 г, опустили кусок льда, имеющий температуру 0 оС. Начальная температура калориметра с водой равна 30 оС. После того, как весь лёд растаял, температура воды и калориметра стала равна 5 оС. Определите массу льда. Ответ округлите до тысячных. Удельная теплоёмкость меди равна     390 , удельная теплоёмкость воды равна 4200  , удельная теплота плавления льда      335 кДж/кг. Потери тепла калориметра считать пренебрежимо малыми.          

С 2



В 17

В сосуд с водой опущена трубка. По трубке через воду пропускают пар при температуре 100 оС. Вначале масса воды увеличивается, но в некоторый момент, масса воды перестаёт увеличиваться, хотя пар по-прежнему пропускают. Первоначальная масса воды 230 г, а в конце её масса  272 г. Какова первоначальная температура воды по шкале Цельсия? Удельная теплоёмкость воды 4200, удельная теплота парообразования воды  2300 кДж/кг.

В 18

В сосуд с водой опущена трубка. По трубке через воду пропускают пар при температуре 100 оС. Вначале масса воды увеличивается, но в некоторый момент, масса воды перестаёт увеличиваться, хотя пар по-прежнему пропускают. Первоначальная масса воды 230 г, а её  первоначальная температура  0 оС. На сколько увеличилась масса воды? Удельная теплоёмкость воды 4200, удельная теплота парообразования воды  2300 кДж/кг. 

С 3

Свинцовую дробь, нагретую до 100 С, в количестве 100 г смешивают с 50 г льда при 0 С. Какой будет  температура  смеси после установления теплового равновесия? Удельная теплоёмкость свинца равна 130 ,  удельная теплота плавления льда 330 кДж/кг. 

12. Внутренняя энергия идеального газа. Изменение внутренней энергии газа

А 1

Внутренняя энергия газа в несжимаемом сосуде определяется главным образом

1) хаотическим движением молекул газа

2) движением всего сосуда с газом

3) взаимодействием сосуда с газом и Земли

4) действием на сосуд с газом внешних сил

А 2  

Внутренняя энергия идеального газа при его охлаждении

  1.  увеличивается
  2.  уменьшается
  3.  увеличивается или уменьшается в зависимости от изменения объема

                      4)  не изменяется

А 3  

Как изменяется внутренняя энергия идеального газа при понижении его температуры?

1) увеличивается

2) уменьшается

3) увеличивается или уменьшается в зависимости от изменения объёма

4) не изменяется

А 4

Внутренняя энергия идеального газа при повышении его температуры

1) увеличивается

2) уменьшается

3) увеличивается или уменьшается в зависимости от изменения объема

4) не изменяется

17

А 5

Внутренняя энергия газа в герметично закрытом сосуде увеличивается при

1) его сжатии

2) его нагревании

3) уменьшении потенциальной энергии сосуда

4) уменьшении кинетической энергии сосуда

А 6

В герметично закрытом сосуде находится одноатомный идеальный газ. Как изменится внутренняя энергия газа при понижении его температуры?

1) увеличится

2) уменьшится

3) увеличится или уменьшится в зависимости от атмосферного давления

4) не изменится

А 7

Меняется ли (и если меняется, то как) энергия хаотического движения молекул газа при его охлаждении от 50 оС до 0 оС?

1) уменьшается, но не до нуля

2) увеличивается

3) уменьшается до нуля

4) не меняется

А 8

Непрерывное тепловое движение молекул газа проявляется в том, что

1) суммарная кинетическая энергия движения молекул никогда не становится равной нулю

2) ни одна молекула никогда не может остановиться

3) средние скорости движения молекул непрерывно изменяются

4) средние скорости движения молекул не меняются с течением времени

А 9

В каком тепловом процессе внутренняя энергия идеального газа постоянной массы не изменяется при переходе его из одного состояния в другое?

1) изобарном

2) изохорном

3) адиабатном

4) изотермическом

А 10

Как изменяется внутренняя энергия идеального газа при его изотермическом расширении?

1) увеличивается

2) уменьшается

3) увеличивается или уменьшается в зависимости от скорости изменения объёма

4) не изменяется

18

А 11

Как изменяется внутренняя энергия идеального газа при его изотермическом сжатии?

1) увеличивается или уменьшается в зависимости от скорости изменения объёма

2) увеличивается

3) уменьшается

4) не изменяется

А 12

Как изменяется внутренняя энергия одноатомного идеального газа при повышении его абсолютной температуры в 2 раза?

1) увеличивается в 4 раза

2) увеличивается в 2 раза

3) уменьшается в 2 раза

4) уменьшается в 4 раза

А 13

Внутренняя  энергия одноатомного идеального газа в закрытом сосуде уменьшилась в 2 раза. При этом температура газа

1) не изменилась

2) повысилась в 4 раза

3) понизилась в 2 раза

4) понизилась в 4 раза

А 14

Как изменяется внутренняя энергия  идеального газа при повышении его  температуры в 2 раза при неизменном давлении?

1) увеличивается в 2 раза

2) уменьшается в 2 раза

3) увеличивается или уменьшается в зависимости от скорости изменения объёма

4) не изменяется

А 15

Как изменяется внутренняя энергия  идеального газа при понижении его  температуры в 2 раза при неизменном давлении?

1) увеличивается в 2 раза

2) уменьшается в 2 раза

3) увеличивается или уменьшается в зависимости от скорости изменения объёма

4) не изменяется

А 16

Как изменяется внутренняя энергия  идеального газа при повышении его  температуры в 2 раза при неизменном объёме?

1) увеличивается в 2 раза

2) уменьшается в 2 раза

3) увеличивается или уменьшается в зависимости от скорости изменения объёма

4) не изменяется

А 17

Как изменяется внутренняя энергия  идеального газа при понижении его  температуры в 2 раза при неизменном объёме?

1) увеличивается в 2 раза

2) уменьшается в 2 раза

3) увеличивается или уменьшается в зависимости от скорости изменения объёма

4) не изменяется

А 18

Гелий в количестве двух моль изотермически сжимают, уменьшая его объем в 2 раза. При этом внутренняя энергия гелия

1) уменьшается в 2 раза

2) не меняется

3) увеличивается в 2 раза   

4) увеличивается в 4 раза

А 19

Гелий в количестве двух моль изотермически расширяется, увеличивая свой объем в 2 раза. При этом его внутренняя энергия

1) увеличивается в 2 раза

2) увеличивается в 4 раза

3) уменьшается в 2 раза

4) не меняется

А 20

Гелий в количестве двух моль изобарно сжимают, уменьшая его объем в 2 раза. При этом внутренняя энергия гелия

1) увеличивается в 4 раза

2) увеличивается в 2 раза

3) не меняется    

4) уменьшается в 2 раза

А 21

Гелий в количестве двух моль изобарно расширяется, увеличивая свой объем в 2 раза. При этом внутренняя энергия гелия

1) увеличивается в 2 раза

2) не меняется

3) уменьшается в 2 раза

4) уменьшается в 4 раза

А 22

Внутренняя энергия идеального газа в процессе, изображенном на рисунке,

1) не изменяется

2) увеличивается   

3) уменьшается  

4) равна нулю                                                              

В 1

Идеальный одноатомный газ находится  в сосуде объёмом 0,6 м3 под давлением 2 кПа. Определите внутреннюю энергию этого газа.

В 2

Идеальный одноатомный газ находится  в сосуде объёмом 0,6 м3. Его внутренняя энергия равна 1,8 кДж. Определите давление газа.

В 3

Идеальный одноатомный газ находится  в сосуде с жесткими стенками объёмом 0,5 м3. При нагревании его давление возросло на  4 кПа. На сколько увеличилась  внутреннюю энергию  газа.

В 4

Идеальный одноатомный газ находится  в сосуде с жесткими стенками объёмом 0,6 м3. При нагревании его давление возросло на 3 кПа. На сколько увеличилась внутренняя энергия газа?

С 1

В цилиндре, закрытом подвижным  поршнем, находится воздух, который мог просачиваться сквозь зазор вокруг поршня. В опыте по изобарному расширению воздуха его объём увеличился в пять раз, а абсолютная температура возросла в 1,2 раза. Во сколько раз изменилась внутренняя энергия воздуха в цилиндре? (Воздух считать идеальным газом).

С 2

В цилиндре, закрытом подвижным  поршнем, находится газ, который мог просачиваться сквозь зазор вокруг поршня. В опыте по сжатию  его объём уменьшился в 6 раз, а абсолютная температура уменьшилась вдвое при неизменном давлении. Во сколько раз изменилась внутренняя энергия газа в цилиндре? (Газ считать идеальным газом).

С 3

В цилиндре, закрытом подвижным  поршнем, находится газ, который мог просачиваться сквозь зазор вокруг поршня. В опыте по изотермическому сжатию газа его объём уменьшился в 2 раз, а давление газа упало в 3 раза. Во сколько раз изменилась внутренняя энергия газа в цилиндре? (Газ считать идеальным газом).

С 4

В цилиндре, закрытом подвижным  поршнем, находится газ. Во время опыта газ сжали и охладили так, что его объём,  уменьшился в 4 раза. Оказалось, однако, что газ мог просачиваться сквозь зазор вокруг поршня, и за время опыта давление газа снизилось в 1,5 раза. Во сколько раз изменилась внутренняя энергия газа в цилиндре? (Газ считать идеальным газом).

С 5

В цилиндре, закрытом подвижным  поршнем, находится воздух. Во время опыта и объём воздуха в цилиндре, и его абсолютную температуру увеличили в 2 раза. Оказалось, однако, что воздух мог просачиваться сквозь зазор вокруг поршня, и за время опыта его давление в цилиндре не изменилось.  Во сколько раз изменилась внутренняя энергия воздуха в цилиндре? (Воздух считать идеальным газом).

С 6

В сосуде с небольшой трещиной находится воздух, который может просачиваться сквозь трещину. Во время опыта давление воздуха в сосуде возросло  в 2 раза, а его абсолютная температура уменьшилась в 4 раза при неизменном объёме. Во сколько раз изменилась внутренняя энергия воздуха в цилиндре? (Воздух считать идеальным газом).

13. Работа в термодинамике

А 1

Объём газа, расширяющегося при постоянном давлении 100 кПа, увеличился на 2 л. Работа, совершенная газом в этом процессе, равна

1) 2000 Дж             2) 20000 Дж

3) 200 Дж            4) 50 МДж

19

А 2

Какую работу совершил аргон массой 0,4 кг при его изобарном нагревании на 10 оС? Молярная масса аргона 0,04 кг/моль.

1) 23,5 кДж

2) 831 Дж

3) 23,5 Дж

4) 8,3 кДж

А 3

Какая работа была совершена при изобарном сжатии 6 моль водорода, если его температура изменилась на 50 К?

1) 1 Дж

2) 69,25 Дж

3) 138,5 Дж

4) 2493 Дж

А 4

В сосуде находится 1 моль гелия. Газ расширился при постоянном давлении и совершил работу 400 Дж. Изменение температуры газа равно

1) 48 К

2) 0,02 К

3) 400 К

4) 1 К

А 5

В сосуде вместимостью 3 под поршнем находится газ. Газ сжимают, прикладывая постоянную силу к поршню, один раз до объёма 2 ,  второй  раз до объёма . Отношение работ, совершенных внешними силами  равно

1) 1                              2) 2

3) 1,5                             4) 3

Геометрический смысл работы (оси рV)

А 6

Какая работа совершается газом при переходе его  из состояния 1 в состояние 2

1) 8 кДж           2) 12 кДж

3) 8 Дж             4) 6 Дж

А 7

В каком из процессов перехода идеального газа из состояния 1 в состояние 2, изображенном на рV-диаграмме (см. рисунок), газ совершает наибольшую работу?

1) А

2) Б

3) В

4) во всех трех процессах газ совершает одинаковую работу

20

А 8

В двух сосудах находится одинаковое количество азота. С газом  в сосудах происходят процессы, показанные           рV-диаграмме. Сравните работы, совершенные над газами в сосудах.

1)        2)   3)       4)

А 9

Идеальный газ переводят из состояния 1 в состояние 3 так, как  показано на графике зависимости давления газа от объёма. Работа, совершенная газом, равна

1)                   2)             3)               4)

А 10

Идеальный газ переводят из состояния 1 в состояние 3 так, как  показано на графике зависимости давления газа от объёма. Работа, совершенная газом, равна

1)               2)          3)                 4)

А 11

Идеальный газ переводят из состояния 1 в состояние 3 так, как  показано на графике зависимости давления газа от объёма. Работа, совершенная газом, равна

1)                   2)            3)            4) 

А 12

Какую работу совершает  газ при переходе из состояния 1 в состояние 3?

1)  10 кДж                2)  20 кДж         3)  30 кДж         4) 40 кДж

А 13

Какую работу совершил одноатомный газ в процессе, изображенном на                   рV-диаграмме?

1) 2,5 кДж            2) 1,5 кДж

3) 3 кДж                4) 4 кДж

А 14

Работа газа за термодина-мический цикл 1-2-3-4 равна

1) 100 кДж           2) 200 кДж

3)  300 кДж         4) 400 кДж

Геометрический смысл работы (оси не рV)

А 15

На рТ-диаграмме показан цикл тепловой машины, у которой рабочим телом является идеальный газ. Сравните абсолютные значения работы газа на участках 1-2 и 3-4

1)

2)

3)

4) для получения ответа не хватает данных

С 1

На рТ-диаграмме показан цикл тепловой машины, у которой рабочим телом является идеальный газ. На каком участке цикла работа газа наибольшая по абсолютной величине?

С 2

На рТ-диаграмме показан цикл тепловой машины, у которой рабочим телом является идеальный газ. На каком из участков  цикла 1-2, 2-3, 3-4,   4-1  работа газа наибольшая по модулю?

С 3

На рТ-диаграмме показан цикл тепловой машины, у которой рабочим телом является идеальный газ. На каком из участков  цикла 1-2, 2-3, 3-4,   4-1  работа газа наименьшая по модулю?

С 4

На VТ-диаграмме показан цикл тепловой машины, у которой рабочим телом является идеальный газ. На каком из участков  цикла 1-2, 2-3, 3-4,   4-1  работа газа наибольшая по модулю?

С 5

На рТ-диаграмме показан цикл тепловой машины, у которой рабочим телом является идеальный газ. Найдите модуль отношения работ газа на участках  3-4 и    1-2.  

21

14. Первое начало термодинамики

А 1

Идеальный газ получил количество теплоты, равное 300 Дж и совершил работу, равную 100 Дж. Как изменилась при этом внутренняя энергия газа?

1) увеличилась на 400 Дж

2) увеличилась на 200 Дж

3) уменьшилась на 400 Дж

4) уменьшилась на 200 Дж

А 2

Идеальному газу сообщили количество теплоты 400 Дж. Газ расширился, совершив работу 600 Дж. Внутренняя энергия газа при этом:

1)

увеличилась на 1000 Дж

2)

увеличилась на 200 Дж

3)

уменьшилась на 1000 Дж

4)

уменьшилась на 200 Дж

А 3

Идеальный газ отдал количество теплоты, равное 300 Дж, а внешние силы совершили над ним работу, равную 100 Дж. При этом внутренняя энергия газа:

1) увеличилась на 400 Дж

2) увеличилась на 200 Дж

3) уменьшилась на 400 Дж

4) уменьшилась на 200 Дж

А 4

Идеальный газ совершил работу, равную 100 Дж, и отдал количество теплоты, равное 300 Дж. При этом внутренняя энергия газа

1) увеличилась на 400 Дж

2) увеличилась на 200 Дж

3) уменьшилась на 400 Дж

4) уменьшилась на 200 Дж

А 5

Идеальный газ получил количество теплоты 100 Дж, и при этом внутренняя энергия газа уменьшилась на 100 Дж. Чему равна работа, совершенная внешними силами над газом?

1) 100 Дж               2) 200 Дж

3) – 200 Дж             4) 0 Дж

А 6

Идеальный газ отдал 300 Дж количества теплоты. При этом внутренняя энергия газа увеличилась на 100 Дж. Чему равна работа, совершенная над газом?

1) 400 Дж

2) 200 Дж

3) – 400 Дж

4) – 200 Дж

А 7

Идеальный газ совершил работу, равную 300 Дж. При этом внутренняя энергия его увеличилась на 300 Дж. В этом процессе газ:

  1.  отдал 600 Дж
  2.  отдал 300 Дж
  3.  получил 600 Дж
  4.  получил 300 Дж

22

А 8

Идеальный газ совершил работу, равную 300 Дж. При этом внутренняя энергия его уменьшилась на 300 Дж. В этом процессе газ:

  1.  отдал 600 Дж
  2.  отдал 300 Дж
  3.  получил 300 Дж
  4.  не отдавал и не получал теплоты

А 9

Газ в сосуде сжали, совершив работу 30 Дж. Внутренняя энергия газа при этом увеличилась на 25 Дж. Следовательно, газ

1) получил извне количество теплоты, равное 5 Дж

2) отдал окружающей среде количество теплоты, равное 5 Дж

3) получил извне количество теплоты, равное 55 Дж

4) отдал окружающей среде количество теплоты, равное 55 Дж

А 10

Внешние силы совершили над идеальным газом работу, равную       500 Дж, и при этом внутренняя энергия газа уменьшилась на 200 Дж. В этом процессе газ:

1) отдал 700 Дж

2) отдал 300 Дж

3) получил 700 Дж

4) получил 300 Дж

А 11

Над идеальным газом внешние силы совершили работу 150 Дж, а его внутренняя энергия уменьшилась на 50 Дж. В этом процессе газ

1) получил количество теплоты 400 Дж

2) получил количество теплоты 200 Дж

3) отдал количество теплоты 200 Дж

4) отдал количество теплоты 100 Дж

В 1

Один моль инертного газа сжали, совершив работу 600 Дж. В результате сжатия температура газа повысилась на 40 оС. Какое количество теплоты отдал газ?

С 1

В сосуде находится под давлением 400 кПа одноатомный идеальный газ, молярная масса которого 0,004 кг/моль. Масса газа 12 г. В результате охлаждения газа давление в сосуде понизилось до 200 кПа. Какой стала температура газа в сосуде после его охлаждения, если отданное им количество теплоты 7,5 кДж?

С 2

В сосуде находится одноатомный идеальный газ, масса которого 12 г, а молярная масса которого 0,004 кг/моль. В начале  давление в сосуде равно  400 кПа. В результате охлаждения газа давление в сосуде понизилось до 200 кПа. Какой была первоначальная температура газа, если отданное им количество теплоты 7,5 кДж?

С 3

В сосуде находится одноатомный идеальный газ, масса которого 12 г, а молярная масса которого 0,004 кг/моль. В начале  давление в сосуде равно  400 кПа при температуре 400 К. Каким станет давление в сосуде после охлаждения,  если отданное газом количество теплоты 7,5 кДж?

С 4

В сосуде находится одноатомный идеальный газ, масса которого 12 г, а молярная масса которого 0,004 кг/моль. В начале  давление в сосуде равно  400 кПа. В результате охлаждения температура газа  понизилась до 200 К. Каков объём сосуда, если отданное им количество теплоты 7,5 кДж?

15. Первое начало термодинамики для изопроцессов

Изотермический процесс

А 1

Идеальный газ переходит изотермически из одного состояния в другое. При увеличении объёма газа

1) ему сообщают некоторое количество теплоты

2) его внутренняя энергия возрастает

3) работа, совершенная внешними телами, положительна

4) давление увеличивается

А 2

Гелий в количестве, равном 1 моль, изотермически сжимают, уменьшая его объем в 2 раза. При этом внутренняя энергия гелия

1) увеличивается за счет работы совершенной внешними силами

2) уменьшается за счет совершения работы газом

3) не меняется, поскольку работа внешних сил равна по модулю работе газа

4) не меняется, поскольку при этом работа внешних сил равна количеству теплоты, отведенной от газа

Изобарный процесс

А 3

Идеальный одноатомный газ совершает переход из состояния 1 в состояние 2 изобарно. Количество теплоты, подведенное к системе в этом процессе, равно 225 кДж. При этом внутренняя энергия газа

1)  увеличилась на 315 кДж

2) уменьшилась на 225 кДж

3) увеличилась на 135 кДж

4) уменьшилась на 90 кДж

А 4

Какое количество теплоты нужно передать молю одноатомного газа, чтобы вдвое увеличить его объём в изобарном процессе, если начальная температура газа Т?

1)                      2)

3)                      4)

В  1

Чему равно изменение внутренней энергии газа, если ему передано количество теплоты 500 Дж, а газ при постоянном давлении 105 Па расширился на 310–3 м3?

В 2

Идеальному одноатомному газу сообщили количество теплоты, равное 1000 Дж. Какая работа будет совершена газом при изобарном расширении?

В 3

Нагреваемый при постоянном давлении идеальный одноатомный газ совершил работу 400 Дж. Какое количество теплоты было передано газу?

В 4

Какое количество теплоты выделится при изобарном охлаждении 80 г гелия с 200 ºС до 100 ºС? Молярная масса гелия 0,004 кг/моль.   Ответ выразите в килоджоулях (кДж) и округлите до целых.

В 5

В цилиндре под поршнем находится идеальный одноатомный газ. Какое количество теплоты получил газ, если при давлении 120 кПа он изобарно расширился с 0,12 м3 до 0,14 м3?

Адиабатный процесс

В 6

В цилиндре под поршнем находится 0,5 кг аргона. Какую работу совершает газ при адиабатном расширении, если его температура понижается на 80 оС? Молярная масса аргона 0,04 кг/моль. 

С 1

Некоторое количество гелия расширяется: сначала адиабатно, а затем изобарно. Конечная температура газа равна начальной. При адиабатном расширении газ совершил работу, равную 4,5 кДж. Какова работа газа за весь процесс?

С 2

В вакууме закреплен горизонтальный цилиндр. В цилиндре находится гелий  в количестве 0,1 моль, запертый

поршнем. Поршень массой 90 г удерживается упорами и может скользить влево вдоль стенок цилиндра без трения. В поршень попадает пуля массой 10 г, летящая горизонтально со скоростью     400 м/с, и застревает в нем. Как изменится температура гелия в момент остановки поршня в крайнем левом положении? Считать, что за время движения поршня газ не успевает обменяться теплом с поршнем и цилиндром.

С 3

В вакууме закреплен горизонтальный цилиндр. В цилиндре находится гелий  в количестве 0,1 моль, запертый

поршнем. Поршень удерживается упорами и может скользить влево вдоль стенок цилиндра без трения. В поршень попадает пуля массой 10 г, летящая горизонтально со скоростью  400 м/с, и застревает в нем. Температура гелия в момент остановки поршня в крайнем левом положении возрастает на      64 К. Какова масса поршня? Считать, что за время движения поршня газ не успевает обменяться теплом с поршнем и цилиндром.

23

В 7

Установите соответствие между описанными в первом столбце особенностями применения первого закона термодинамики  к различным изопроцессам и названием изопроцесса.

ОСОБЕННОСТИ ПРИМЕНЕНИЯ ПЕРВОГО ЗАКОНА ТЕРМОДИНАМИКИ

НАЗВАНИЕ ПРОЦЕССА

А) Все переданное газу количество теплоты идет на совершение работы, а внутренняя энергия газа остаётся неизменной.

Б) Изменение внутренней энергии газа происходит только за счет совершения работы, так как теплообмен с окружающими  телами отсутствует.

1) изотермический

2) изобарный

3) изохорный

4) адиабатный

А

Б

Получившуюся последовательность цифр перенесите в бланк ответов (без пробелов и каких-либо символов)

В 8

Установите соответствие между описанными в первом столбце особенностями применения первого закона термодинамики  к различным изопроцессам и названием изопроцесса.

ОСОБЕННОСТИ ПРИМЕНЕНИЯ ПЕРВОГО ЗАКОНА ТЕРМОДИНАМИКИ

НАЗВАНИЕ ПРОЦЕССА

А) Все переданное газу количество теплоты идет на изменение  внутренней энергии газа.

Б) Изменение внутренней энергии газа происходит только за счет совершения работы, так как теплообмен с окружающими  телами отсутствует.

В) Все переданное газу количество теплоты идет на совершение работы, а внутренняя энергия газа остаётся без изменения

1) изотермический

2) изобарный

3) изохорный

4) адиабатный

А

Б

В

Получившуюся последовательность цифр перенесите в бланк ответов (без пробелов и каких-либо символов)

24

В 9

Установите соответствие между описанными в первом столбце особенностями применения первого закона термодинамики  к различным изопроцессам и названием изопроцесса.

ОСОБЕННОСТИ ПРИМЕНЕНИЯ ПЕРВОГО ЗАКОНА ТЕРМОДИНАМИКИ

НАЗВАНИЕ ПРОЦЕССА

А) Все переданное газу количество теплоты идет на изменение  внутренней энергии газа.

Б) Все переданное газу количество теплоты идет на совершение работы, а внутренняя энергия газа остаётся без изменения.

В) Изменение внутренней энергии газа происходит только за счет совершения работы, так как теплообмен с окружающими  телами отсутствует.

1) адиабатный

2) изобарный

3) изотермический

4) изохорный

А

Б

В

Получившуюся последовательность цифр перенесите в бланк ответов (без пробелов и каких-либо символов)

В 10

Установите соответствие между описанными в первом столбце особенностями применения первого закона термодинамики  к различным изопроцессам и названием изопроцесса.

ОСОБЕННОСТИ ПРИМЕНЕНИЯ ПЕРВОГО ЗАКОНА ТЕРМОДИНАМИКИ

НАЗВАНИЕ ПРОЦЕССА

А) Все переданное газу количество теплоты идет на совершение работы, а внутренняя энергия газа остаётся без изменения

Б) Все переданное газу количество теплоты идет на изменение  внутренней энергии газа.

В) Изменение внутренней энергии газа происходит только за счет совершения работы, так как теплообмен с окружающими  телами отсутствует

1) адиабатный

2) изотермический

3) изобарный

4) изохорный

А

Б

В

Получившуюся последовательность цифр перенесите в бланк ответов (без пробелов и каких-либо символов)

В 11

Установите соответствие между описанными в первом столбце особенностями применения первого закона термодинамики  к различным изопроцессам и названием изопроцесса.

ОСОБЕННОСТИ ПРИМЕНЕНИЯ ПЕРВОГО ЗАКОНА ТЕРМОДИНАМИКИ

НАЗВАНИЕ ПРОЦЕССА

А) Изменение внутренней энергии газа происходит только за счет совершения работы, так как теплообмен с окружающими  телами отсутствует.

Б) Все переданное газу количество теплоты идет на изменение  внутренней энергии газа.

В) Все переданное газу количество теплоты идет на совершение работы, а внутренняя энергия газа остаётся без изменения

1) изохорный

2) адиабатный

3) изотермический

4) изобарный

А

Б

В

Получившуюся последовательность цифр перенесите в бланк ответов (без пробелов и каких-либо символов)

В 12

Установите соответствие между описанными в первом столбце особенностями применения первого закона термодинамики  к различным изопроцессам и названием изопроцесса.

ОСОБЕННОСТИ ПРИМЕНЕНИЯ ПЕРВОГО ЗАКОНА ТЕРМОДИНАМИКИ

НАЗВАНИЕ ПРОЦЕССА

А) Все переданное газу количество теплоты идет на совершение работы, а внутренняя энергия газа остаётся без изменения

Б) Все переданное газу количество теплоты идет на изменение  внутренней энергии газа.

В) Изменение внутренней энергии газа происходит только за счет совершения работы, так как теплообмен с окружающими  телами отсутствует.

1) изотермический

2) изобарный

3) изохорный

4) адиабатный

А

Б

В

Получившуюся последовательность цифр перенесите в бланк ответов (без пробелов и каких-либо символов)

16. Первое начало термодинамики (графические задачи)

А 1

На  рV-диаграмме  показан процесс изменения состояния идеального одноатомного газа. Газ совершает работу, равную   3 кДж. Количество теплоты, полученное газом, равно

1) 1 кДж

2) 3 кДж

3) 4 кДж

4) 7 кДж

А 2

На  графике  показана зависимость давления одноатомного идеального газа от объёма. Газ совершает работу, равную   4 кДж. Количество теплоты, полученное газом при переходе из состояния 1 в состояние 2, равно

1) 1 кДж

2) 4 кДж

3) 5 кДж

4) 7 кДж

В 1

На рисунке показан процесс изменения состояния идеального газа. Внешние силы совершили над газом работу, равную 5·10Дж. Какое количество теплоты отдает газ в этом процессе? Ответ выразите в килоджоулях (кДж).

А 3

Идеальный одноатомный газ совершает переход из состояния 1 в состояние 2 (рис.). При этом подводится количество тепло-ты, равное

1) 200 кДж

2) 300 кДж

3) 500 кДж

4) 700 кДж

25

А 4

Идеальный одноатомный газ совершает переход из состояния 1 в состояние 2 (рис.).  В этом процессе от системы было отведено  количество теплоты, равное

1) 150 кДж

2) 225 кДж

3) 375 кДж

4) 525 кДж

А 5

Идеальный одноатомный газ совершает переход из состояния 1 в состояние 2 (рис.). При этом количество теплоты подведен-ное к системе  равно

1) 150 кДж

2) 225 кДж

3) 375 кДж

4) 525 кДж

А 6

Идеальный одноатомный газ совершает переход из состояния 1 в состояние 2, согласно представленному рисунку. При этом внутренняя энергия газа

1) уменьшилась на 315 кДж

2) увеличилась на 225 кДж

3) уменьшилась на 135 кДж       

4) увеличилась на 90 кДж

А  7

На рТ-диаграмме (см. рисунок) показан процесс изменения состояния идеального одноатомного газа. Газ отдает 25 кДж теплоты. Работа внешних сил равна

1)

0 кДж

2)

25 кДж

3)

50 кДж

4)

100 кДж

26

А 8

На рТ-диаграмме (см. рисунок) показан процесс изменения состояния идеального одноатомного газа. Газ отдает 50 кДж теплоты. Работа внешних сил равна

1)

0 кДж

2)

25 кДж

3)

50 кДж

4)

100 кДж

А 9

На рТ-диаграмме показан процесс изменения состояния  идеального газа. Газ совершает   работу, равную 3 кДж. Начальный объём газа 10-3 м3. Количество теплоты, полученное газом

1) 1 кДж

2) 3 кДж

3) 4 кДж

4)  7 кДж

А 10

На графике показана зависимость давления  идеального одноатомного газа от температуры. Газ совершает   работу, равную 4 кДж. Количество теплоты, полученное газом

1) 1 кДж

2) 7 кДж

3) 3 кДж

4)  4 кДж

А  11

На VТ-диаграмме (см. рисунок) показан процесс изменения состояния идеального одноатомного газа. Газ отдает 50 кДж теплоты. Работа внешних сил равна

1)

0 кДж

2)

25 кДж

3)

50 кДж

4)

100 кДж

А 12

На ТV-диаграмме показан процесс изменения состояния  идеального газа. Газ совершает   работу, равную 3 кДж. Начальное давление газа 106 Па. Количество теплоты, полученное газом

1) 1 кДж                                         2) 3 кДж

3) 4 кДж                                         4) 7 кДж

А 13

На ТV-диаграмме показан процесс изменения состояния  идеального газа. Газ получает    50 кДж теплоты. Работа, совершенная газом, равна

1) 0 кДж                                            2) 25 кДж

3) 50 кДж                                         4) 100 кДж

А 14

На ТV-диаграмме показан процесс изменения состояния  идеального газа. Газ отдает        20 кДж теплоты. Работа внешних сил  равна

1) 0 кДж                                            2) 20 кДж

3) 50 кДж                                          4) 100 кДж

А 15

На Тр-диаграмме показан процесс изменения состояния  идеального газа. Газ совершает   работу, равную 3 кДж. Начальный объём газа 10-3 м3. Количество теплоты, полученное газом

1) 0 кДж                                         2) 1 кДж

3) 3 кДж                                         4) 4 кДж

А 16

На Тр-диаграмме показан процесс изменения состояния  идеального газа. Газ получает 100 кДж теплоты. Работа, совершенная газом,   равна

1) 0 кДж                                            2) 25 кДж

3) 50 кДж                                          4) 100 кДж

А 17

На Тр-диаграмме показан процесс изменения состояния  идеального газа. Внутренняя энергия газа уменьшилась на     15 кДж теплоты. Количество теплоты, отданное газом

1) 0 кДж                                            2) 15 кДж

3) 30 кДж                                          4) 60 кДж

А 18

На графике показана зависимость температуры от давления  идеального одноатомного газа. Внутренняя энергия газа увеличилась на      10 кДж. Количество теплоты, полученное газом

1) 0 кДж                                            2) 10 кДж

3) 20 кДж                                          4) 40 кДж

А 19

На  рисунке  представлен график зависимости давления идеального одноатомного газа от объёма при адиабатном расширении. Газ совершает работу, равную   20 кДж. Внутренняя энергия газа при этом

1) не изменилась

2)  увеличилась на 20 кДж

3) уменьшилась на 20 кДж

4) уменьшилась на 40 кДж

27

А 20

На  рисунке  представлен график зависимости давления идеального одноатомного газа от объёма при его адиабатном сжатии. Внешние силы  совершили работу, равную      30 кДж. Внутренняя энергия газа при этом

1) не изменилась

2)  увеличилась на 30 кДж

3) уменьшилась на 30 кДж

4) уменьшилась на 60 кДж

А 21

На рисунке приведен график зависимости давления одноатомного идеального газа от его объёма. Внутренняя энергия газа увеличилась на 300 кДж

1) 0 кДж                                         2) 100 кДж

3) 200 кДж                                      4) 500 кДж

А 22

На рисунке приведен график зависимости давления одноатомного идеального газа от его объёма. Газ получил 500 кДж

1) не изменилась                               2) увеличилась на 100 кДж

3) уменьшилась на 100 кДж            4) увеличилась на 300 кДж

А 23

На рисунке приведен график зависимости давления одноатомного идеального газа от его объёма. Газ отдал 500 кДж

1) не изменилась                               2) увеличилась на 100 кДж

3) уменьшилась на 300 кДж           4) увеличилась на 500 кДж

28

А 24

На рисунке приведен график зависимости давления одноатомного идеального газа от его объёма. Газ получил 300 кДж

1) не изменилась                               2) увеличилась на 100 кДж

3) уменьшилась на 200 кДж            4) увеличилась на 300 кДж

А 25

На рисунке приведен график зависимости давления одноатомного идеального газа от его объёма. Газ отдал 250 кДж

1) не изменилась                               2) уменьшилась  на 50 кДж

3) уменьшилась на 150 кДж           4) уменьшилась на 350 кДж

В 2

В сосуде находится 1 моль одноатомного идеального газа. Какое количество теплоты получил газ в процессе, изображенном на                   рV-диаграмме (см. рисунок)? Ответ выразите в килоджоулях (кДж) и округлите до целых.  

А 26

На рисунке приведен график зависимости объема идеального одноатомного газа от давления в процессе 1 – 2. Внутренняя энергия газа при этом увеличилась на 300 кДж. Количество теплоты, сообщенное газу в этом процессе, равно

1)

0 кДж

2)

100 кДж

3)

200 кДж

4)

500 кДж

А 27

График зависимости давления от объема для циклического процесса изображен на рисунке. В этом процессе газ 

1) совершает положительную работу

2) совершает отрицательную работу

3) не получает энергию от внешних источников

4) не отдает энергию внешним телам

В 3

Первое начало термодинамики для изопроцессов

С 1

Один моль идеального одноатомного газа сначала  изотермически расширился      1=300 К). Затем газ охладили, понизив давление в 3 раза (см. рисунок). Какое количество теплоты отдал  газ на участке         2 3?

  

С 2

Над идеальным одноатомным газом в количестве вещества 2 моль совершили процесс 1-2-3 (рис.) Температура газа в состоянии 1 равна 280 К, участок 1 - 2 является изотермой. Какое количество теплоты было передано газу на участке 2 – 3?

С 3

Над идеальным одноатомным газом в количестве вещества     1 моль совершили процесс        1-2-3-1 (рис.). Температуру газа в состоянии 2 составила 320 К, участок 1-2 является изопроцес-сом. Какое количество теплоты отдал газ на участке 3- 1?

С 4

Идеальный одноатомный газ в количестве       2 моль   сначала  изотермически сжали   (Т1=250 К).  Затем  газ нагрели, повысив давление в 3 раза (см. рисунок). Какое количество теплоты получил газ на участке    2-3?

  

С 5

Идеальный одноатомный газ в количестве 2 моль   сначала  изотермически расширился (Т1=400 К).  Затем  газ изобарно нагрели, повысив температуру в 2 раза (см. рисунок). Какое количество теплоты получил газ на участке 2-3?

  

С 6

Один моль идеального одноатомного газа сначала нагрели, а затем охладили до первоначальной температуры 300 К, уменьшив давление в 3 раза (см. рисунок). Какое количество теплоты сообщено газу на участке 1-2?

  

С 7

10 моль идеального одноатомного газа охладили, уменьшив давление в 3 раза. Затем газ нагрели до первоначальной температуры 300 К (см. рисунок). Какое количество теплоты сообщено газу на участке 2 3?

  

29

С 8

Идеальный одноатомный газ в количестве двух моль сначала изотермически расширился  (Т1=400 К). Затемгаз изобарно нагрели, повысив температуру в 3 раза (см. рисунок). Какое количество теплоты получил газ на участке 2-3?

С 9

Два моль одноатомного идеального газа сначала охладили, а затем нагрели до первоначальной температуры 400 К, увеличив объем газа в 4 раза (см. рисунок). Какое количество теплоты отдал газ на участке 1 2?

 

С 10

Одноатомного идеальный газ в количестве 10 моль сначала охладили, уменьшив давление в 3 раза, а затем нагрели до первоначальной температуры 300 К. Какое количество теплоты получил газ на участке 2 3?

 

С 11

Один  моль одноатомного идеального газа сначала нагрели, а затем охладили до первоначальной температуры 300 К, уменьшив давление  в 3 раза (см. рисунок). Какое количество теплоты сообщено  газу на участке 1 2?

 

30

С 12

Один  моль идеального одноатомного газа сначала изотермически расширился  (Т1=300 К).  Затем газ охладили, понизив давление в 3 раза  (см. рисунок). Какое количество теплоты отдал  газ на участке 2-3?

 

С 13

1 моль идеального одноатомного газа сначала охладили, а затем нагрели до первоначальной температуры 300 К, увеличив объем газа в 3 раза (см. рисунок). Какое количество теплоты отдал газ на участке 1 2?

 

С 14

Над идеальным одноатомным газом в количестве вещества   1,4 моль совершили процесс    1-2-3-1 (рис.). Какое количество теплоты подведено к системе на участке 1 - 2, если температура газа в точке 3 равна 580 К?

С 15

Над идеальным одноатомным газом в количестве вещества   0,8 моль совершили процесс       1 - 2 - 3 - 1 (рис.). Какое количество теплоты газ отдает на участке 3 - 1, если температура газа в точке 1 равна 280 К?

С 16

Над идеальным одноатомным газом в количестве вещества  1,6 моль совершают процесс    1-2-3-1 (рис.). Какое количество теплоты газ отдает на участке  2-3, если температура газа в точке 1 равна 220 К?                 

С 17

Идеальный одноатомный газ в количестве двух моль сначала изотермически расширился  (Т1=300 К).  Затем газ охладили, понизив давление в 2 раза  (см. рисунок). Какое количество теплоты отдал  газ на участке 2-3?

 

С 18

Два  моль идеального одноатомного газа охладили, уменьшив давление  в 4 раза.  Затем газ  нагрели до первоначальной температуры 400 К, (см. рисунок). Какое количество теплоты сообщено  газу на участке 2-3?

 

С 19

1 моль идеального одноатомного газа сначала нагрели, а затем охладили до первоначальной температуры 300 К, уменьшив давление  в 3 раза (см. рисунок). Какое количество теплоты сообщено  газу на участке 1 2?

 

С 20

10  моль идеального одноатомного газа охладили, уменьшив давление  в 3 раза.  Затем газ  нагрели до первоначальной температуры 300 К, (см. рисунок). Какое количество теплоты сообщено  газу на участке 2-3?

Работа

С 21

Идеальный одноатомный газ сжимают  сначала  адиабатно, а затем изобарно. Конечная температура газа равна начальной (см. рисунок). За весь процесс 1-2-3 внешние силы совершили работу, равную 5 кДж. Какова работа внешних сил при изобарном сжатии газа?

 

С 22

Идеальный одноатомный газ сжимают  сначала  адиабатно, а затем изобарно. Конечная температура газа равна начальной (см. рисунок). При адиабатном сжатии газа  внешние силы совершили работу, равную        3 кДж. Какова работа внешних сил  за весь процесс 1-2-3?

С 23

Идеальный одноатомный газ сжимают  сначала  адиабатно, а затем изобарно. Конечная температура газа равна начальной (см. рисунок). При адиабатном сжатии газа  внешние силы совершили работу, равную        6 кДж. Какова работа внешних сил  за весь процесс 1-2-3?

С 24

Идеальный одноатомный газ расширяется сначала  адиабатно, а затем изобарно. Конечная температура газа равна начальной (см. рисунок). За весь процесс 1-2-3 газом   совершена работу, равная 5 кДж. Какую работу совершил газ при изобарном  расширении?

  

С 25

Идеальный одноатомный газ расширяется сначала  адиабатно, а затем изобарно. Конечная температура газа равна начальной (см. рисунок). За весь процесс 1-2-3 газом   совершена работу, равная 5 кДж. Какую работу совершает газ при адиабатном  расширении?

   

С26

Идеальный одноатомный газ расширяется сначала  адиабатно, а затем изобарно. Конечная температура газа равна начальной (см. рисунок). При адиабатном расширении газ   совершил работу, равную 3 кДж. Какова работа газа  за весь процесс 1-2-3?

  

Первое начало термодинамики                                                                                 31

С 27

Какое количество теплоты подведено к двум моль одноатомного идеального газа при осуществлении процесса    1-2-3, если начальная температура его была равна   300 К?

С 28

Какое количество теплоты подведено к двум моль одноатомного идеального газа при осуществлении процесса    1-2-3, если конечная температура его была равна      Т3 = 600 К?

С 29

Какое количество теплоты подведено к двум моль одноатомного идеального газа при осуществлении процесса    1-2-3, если в состоянии 2 температура его была равна      Т2 = 400 К?

С 30

Какое количество теплоты подведено к двум моль одноатомного идеального газа при      осуществлении процесса          1-2-3, если начальный объем его равен V1 = 1 л, а давление равно          р1 = 100 кПа?

С 31

Какое количество теплоты подведено к двум моль одноатомного идеального газа при      осуществлении процесса          1-2-3, если конечный объем его равен V3 = 6 л, а давление равно          р3 = 200 кПа?

32

С 32

Один моль одноатомного идеального газа совершает процесс 1-2-3.  На участке 2-3 к газу подводят      2,5 кДж теплоты,  Т0=300 К. Найдите отношение количества  подведенного к газу теплоты Q123 к  работе А123, совершаемой газом в ходе процесса.

С 33

Один моль одноатомного идеального газа переходит из состояния 1 в состояние 2 в соответствии с графиком зависимости его объёма  V от температуры Т (Т0=100 К). На участке 2-3 к газу подводят      2,5 кДж теплоты. Найдите отношение работы газа  А123 ко всему количеству подведенной к газу теплоты Q123.  

С 34

Один моль одноатомного газа совершает процесс 1-2-3 (см. рисунок, где Т0=300 К). На участке 2-3 к газу подводят    2,5 кДж теплоты. Найдите отношение полного подведенного к газу количества теплоты Q123 к  работе А123, совершенной газом в ходе процесса.

С 35

Один моль одноатомного газа совершает процесс 1-2-3 (см. рисунок, где       Т0=100 К). На участке 2-3 к газу подводят 2,5 кДж теплоты. Найдите отношение работы   А123, совершаемой газом к  количеству поглощенной газом  теплоты Q123.

 

17. КПД тепловой машины

КПД, количество теплоты, полученное от нагревателя и отданное холодильнику

А 1

Рабочее тело тепловой машины получило 70 кДж теплоты. При этом холодильнику передано 52,5 кДж теплоты. КПД такой машины

1) 1,7 %

2) 17,5 %

3) 25 %

4) >100 %

А 2

Тепловой двигатель получает за цикл от нагревателя 200 Дж теплоты и отдаёт холодильнику 150 Дж. КПД двигателя равен

1) 25 %

2) 33 %

3) 67 %

4) 75 %

А 3

Тепловая машина за цикл работы получает от нагревателя количество теплоты, равное 100 Дж,   и отдаёт холодильнику количество теплоты, равное 40 Дж. Чему равен КПД  тепловой машины?

1) 40 %

2) 60 %

3) 29 %

4) 43 %

А 4

Тепловой двигатель за цикл получает от нагревателя количество теплоты, равное 3 кДж и отдает холодильнику количество теплоты, равное 2,4 кДж. КПД двигателя равен

1)

20%

2)

25%

3)

80%

4)

120%

А  5

За цикл работы идеального теплового двигателя рабочему телу от
нагревателя было передано количество теплоты 80 Дж, а холодильнику от рабочего тела - количество теплоты 60 Дж. КПД теплового двигателя   равен            

1) 25%                       2) 33%

3) 67%                      4) 75%

А 6

Тепловой двигатель за цикл работы отдаёт холодильнику количество теплоты, равное 66 Дж, получая при этом от нагревателя количество теплоты, равное 99 Дж.  КПД двигателя с точностью до сотых равен:

1) 0,20

2) 0,33

3) 0,50

4) 0,67

А 7

Тепловая машина с КПД 40 % получает за цикл от нагревателя         100 Дж. Какое количество теплоты машина отдает за цикл холодильнику?

  1.  40 Дж                 2) 60 Дж             3) 100 Дж                  160 Дж

А 8

Тепловая машина с КПД, равным 60 %, за некоторое время получает от  нагревателя количество теплоты, равное 50 Дж. Какое количество теплоты машина отдаёт за это время окружающей среде?

1) 20 Дж                  2) 30 Дж

3) 50 Дж                  4) 80 Дж

А 9

Тепловая машина с КПД 40 % за цикл работы отдаёт холодильнику количество теплоты, равное 60 Дж. Какое количество теплоты за цикл получает машина от нагревателя?

1) 250 Дж

2) 150 Дж

3) 100 Дж

4) 24 Дж

А 10

Тепловая машина с КПД 50 % за цикл работы отдаёт холодильнику количество теплоты, равное 100 Дж. Какое количество теплоты за цикл получает машина от нагревателя?

1) 200 Дж

2) 150 Дж

3) 100 Дж

4) 50 Дж

А 11

Тепловая машина с КПД 60 % за цикл работы отдаёт холодильнику количество теплоты, равное 100 Дж. Какое количество теплоты за цикл получает машина от нагревателя?

1) 600 Дж                2) 250 Дж

3) 150 Дж               4) 60 Дж

А 12

Тепловой двигатель за цикл работы получает от нагревателя количество теплоты на 33% больше по сравнению с тем количеством теплоты, которое отдает холодильнику. КПД теплового двигателя с точностью до целых равен:

1) 25%

2) -33%

3) 49%

4) 67%

КПД, механическая работа и количество теплоты, полученное от нагревателя

А 13

При работе двигателя внутреннего сгорания автомобиля энергия, выделившаяся при  сгорании топлива, …

1) полностью превращается в механическую энергию автомобиля

2) частично превращается в механическую энергию автомобиля

3) полностью превращается во внутреннюю энергию выхлопных газов

4) полностью превращается в кинетическую энергию выхлопных газов

А 14

Как связаны между собой модули количеств теплоты, передаваемых за цикл в ходе теплообмена между рабочим телом и нагревателем , между рабочим телом и холодильником и работы , которую рабочее тело совершает за цикл

1) 

2)

3)

4)

А 15

Чему равен коэффициент полезного действия двигателя внутреннего сгорания, если полученное им количество теплоты равно 100 кДж, а полезная работа составляет 20 кДж?

1) 5 %

2) 20 %

3) 50 %

4) 80 %

А 16

Чему равен коэффициент полезного действия паровой турбины, если полученное ею количество теплоты равно 1000 МДж, а полезная работа составляет 400 МДж?

1) 4 %

2) 25 %

3) 40 %

4) 60 %

33

А 17

Тепловая машина за цикл получает от нагревателя 50 Дж  и совершает полезную работу, равную 100 Дж. Чему равен КПД  тепловой машины?

1) 200 %

2) 67 %

3) 50 %

4) такая машина невозможна

А 18

Тепловая машина с КПД 25 % за цикл работы получает от нагревателя  100 Дж. Какую полезную работу машина совершает за один цикл?

1) 400 Дж

2) 250 Дж

3) 75 Дж

4) 25 Дж

А 19

Тепловая машина с КПД 40 % за цикл работы получает от нагревателя количество теплоты, равное 100 Дж. Какую полезную работу машина совершает за один цикл?

1) 40 Дж

2) 60 Дж

3) 100 Дж

4) 160 Дж

А 20

Тепловая машина с КПД 60 % за цикл работы получает от нагревателя количество теплоты, равное 100 Дж. Какую полезную работу машина совершает за один цикл?

1) 40 Дж

2) 60 Дж

3) 100 Дж

4) 160 Дж

А 21

Тепловая машина с КПД 75 % за цикл работы получает от нагревателя количество теплоты, равное 60 Дж. Какую полезную работу машина совершает за один цикл?

1) 80 Дж

2) 50 Дж

3) 45 Дж

4) 15 Дж

А 22

Тепловой двигатель с КПД, равным 40%, за цикл совершает полезную работу 200 Дж. Какое количество теплоты рабочее тело двигателя получает от нагревателя за цикл работы?

1) 80 Дж

2) 120 Дж

3) 300 Дж

4) 500 Дж

КПД, механическая работа и количество теплоты, отданное холодильнику

А 23

Тепловая машина за цикл совершает работу 50 Дж, и отдаёт холодильнику количество теплоты, равное 100 Дж. Чему равен КПД  тепловой машины?

1) 100 %

2) 50 %

3)  33%

4) 67 %

А 24

Тепловой двигатель за цикл работы отдает холодильнику количество теплоты 120 Дж, при этом совершает полезную работу 20 Дж. КПД двигателя с точностью до сотых, равен:        

1) 0,14                   2) 0,17

3) 0,20                   4) 0,25

А 25

Тепловая машина с КПД 20 % за цикл работы отдаёт холодильнику количество теплоты, равное 80 Дж. Какую полезную работу машина совершает за цикл?

1) 100 Дж              2) 64 Дж

3) 20 Дж               4) 16 Дж

34

КПД, температуры нагревателя и холодильника

А 26

Вычислите максимальное значение коэффициента полезного действия тепловой машины, если температура нагревателя 127 оС, а температура холодильника 27 оС.

1) 25 %

2) 75 %

3) 78,7 %

4) 21,3 %

А 27

Максимальное значение КПД тепловой машины с температурой нагревателя 227 оС и температурой холодильника 27 оС равно

1) 100 %

2) 88 %

3) 60 %

4) 40 %

А 28

Каков максимальный КПД, который может иметь тепловой двигатель с температурой нагревателя 727оС и температурой холодильника 27оС?

1) 95 %

2) 70 %

3) >100 %

4) 1 %

А 29

Горячий пар поступает в турбину при температуре 500 оС, а выходит из неё при температуре 30 оС. Каков КПД турбины? Паровую турбину считать идеальной тепловой машиной.

1) 1 %

2) 61 %

3) 94 %

4) 100 %

А 30

В тепловой машине температура нагревателя 600 К, а температура холодильника на 200 К меньше, чем у нагревателя. Максимально возможный КПД машины равен

1) ¾                       2) 2/3

3) ½                       4) 1/3

А 31

У идеальной тепловой машины Карно температура холодильника равна 300 К. Какой должна быть температура её нагревателя, чтобы КПД машины был равен 40 %?

1) 1200 К

2) 800 К

3) 600 К

4) 500 К

А 32

КПД тепловой машины 45 %. Рассчитайте температуру нагревателя, если температура холодильника 2 оС.

1) 500 оС

2) 500 К

3) 45 К

4) 5 К

А 33

КПД идеального теплового двигателя 40 %. Чему равна температура нагревателя, если температура холодильника 27 оС

1) 180 К

2) 500 К

3) 750 К

4) 1080 К

А 34

КПД идеального теплового двигателя 30 %. Какова температура нагревателя, если температура холодильника 20 оС? Ответ округлите до целых

1) 419 оС

2) 419 К

3) 29 оС

4) 10 К

Остальные задачи

А 35

Температура нагревателя идеальной тепловой машины 425 К, а температура холодильника 300 К. Двигатель получил от нагревателя количество теплоты 40 кДж. Какую работу совершило рабочее тело?

1) 3 кДж               2) 11,8 кДж

3) 16,7 кДж          4) 97 Дж

А 36

Тепловая машина имеет КПД 25 %. Средняя мощность передачи теплоты холодильнику в ходе её работы составляет 3 кВт. Какое количество теплоты получает рабочее тело машины от нагревателя за 10 с?

1) 0,4 Дж                 2) 40 Дж

3) 400 Дж                4) 40 кДж

А 37

У двух идеальных тепловых машин температуры холодильников в 3 раза ниже температур нагревателей соответствующих машин, но при этом не равны между собой. У машины с меньшей температурой холодильника КПД

1) выше в 3 раза, чем у машины с большей температурой  холодиль-ника

2) ниже в 3 раза, чем у машины с большей температурой холодиль-ника

3) также как у машины с большей температурой холодильника равен 67%

4) ничего определенного утверждать нельзя, если неизвестны температуры нагревателей

А 38

У двух идеальных тепловых машин температуры холодильников отличаются в 2 раза, а температуры нагревателей одинаковы. Выберите верное утверждение. У машины с большей температурой холодильника КПД всегда

1) в 2 раза больше, чем у машины с меньшей температурой холодильника

2) в 2 раза меньше, чем у машины с меньшей температурой холодильника

3) больше, чем у машины с меньшей температурой холодильника
4) меньше, чем у машины с меньшей температурой холодильника

А 39

Идеальный тепловой двигатель имеет определенный КПД. Рассматривается два типа изменений, вносимых в работу двигателя. Один раз температура нагревателя поднимается на величину  при неизменной температуре холодильника, второй раз температуру холодильника понижают на величину , оставляя прежней температуру нагревателя. В каком случае КПД новой тепловой машины будет выше

  1.  в первом
  2.  во втором
  3.  в обоих случаях одинаков
  4.  ответ зависит от начальных температур нагревателя и холодильника

А 40

Сравните КПД тепловых машин, работающих по циклам

1)

2)

3)

4) сравнить невозможно

А

В

А 41

На рисунке представлены циклы двух тепловых машин: серым закрашен цикл, соответствующий машине А, белым - машине В. Согласно приведенным на рисунке циклам, для КПД машин справедливым является утверждение, что

1)     2)       3)          4) 

А 42

Сравните КПД тепловых машин, в каждой из которых рабочее тело меняет давление и свой объём циклически. Цикл, соответствующий машине А, на рисунке показан белым, а цикл, соответствующей машине В - серым

1)

2) 

3)

4) 

35

18. КПД замкнутого цикла

С 1

Рассчитайте КПД тепловой машины, использующей в качестве рабочего тела одноатомный идеальный газ и работающей по циклу, изображенному на рисунке.

19. Газы в теплоизолированном сосуде

С 1

Теплоизолированный сосуд объемом  разделен пористой  перегородкой на 2 равные части. Атомы гелия могут свободно проникать через поры в перегородке, а атомы аргона - нет. В начальный момент в  одной части сосуда находится  моль  гелия, а в другой  моль  аргона. Температура  гелия равна , а   аргона . Определите температуру гелия после установления равновесия в системе.

С 2

Теплоизолированный сосуд объемом  разделен пористой  перегородкой на 2 равные части. Атомы гелия могут свободно проникать через поры в перегородке, а атомы аргона - нет. В начальный момент в  одной части сосуда находится  гелий массой , а в другой аргон массой . Средняя квадратичная скорость атомов гелия равна средней квадратичной скорости атомов аргона и составляет . Определите температуру гелий-аргоновой смеси после установления равновесия в системе. Молярная масса гелия 0,004 кг/моль, а аргона 0,04 кг/моль.

С 3

Теплоизолированный сосуд разделен теплопроводной неподвижной перегородкой на две части одинакового объема. В одной части находится  моль гелия, а в другой -  моль аргона. В начальный момент времени средняя квадратичная скорость атомов аргона в 2 раза больше средней квадратичной скорости атомов гелия. Определите отношение давления гелия к давлению аргона после установления теплового равновесия. Молярная масса гелия             0,004 кг/моль, а аргона 0,04 кг/моль. 

36

С 4

Теплоизолированный сосуд объемом  разделен перегородкой на две равные части. В одной части сосуда находится гелий массой , а в другой аргон массой . Средняя квадратичная скорость атомов гелия равна средней квадратичной скорости атомов аргона и составляет . Определите парциальное давление гелия после удаления перегородки. Молярная масса гелия               0,004 кг/моль, а аргона 0,04 кг/моль.

С 5

Теплоизолированный сосуд  объёмом  разделён теплоизолирующей перегородкой на две равные части. В одной части сосуда находится  моль гелия, а в другой – такое же количество аргона. Температура гелия , а температура аргона . Определите парциальное давление аргона в сосуде после удаления перегородки.

С 6

Теплоизолированный сосуд объемом  разделен пористой  перегородкой на 2 равные части. В начальный момент в  одной части сосуда находится  гелий массой , а в другой аргон массой . Атомы гелия могут свободно проникать через поры в перегородке, а атомы аргона - нет. Начальная температура  гелия равна температуре аргона: , Определите внутреннюю энергию  газа, оставшегося в той части сосуда, где первоначально находился гелий,  после установления равновесия в системе. Молярная масса гелия 0,004 кг/моль, а аргона  0,04 кг/моль.     

С 7

Теплоизолированный сосуд объемом  разделен пористой  перегородкой на 2 равные части. В начальный момент в  одной части сосуда находится  гелий массой , а в другой аргон массой . Атомы гелия могут свободно проникать через поры в перегородке, а атомы аргона - нет. Начальная температура  гелия равна температуре аргона: , Определите внутреннюю энергию   гелий-аргоновой смеси  после установления равновесия в системе. Молярная масса гелия 0,004 кг/моль, а аргона  0,04 кг/моль. 

С 8

Теплоизолированный сосуд объемом  разделен пористой  перегородкой на 2 равные части. Атомы гелия могут свободно проникать через поры в перегородке, а атомы аргона - нет. В начальный момент в  одной части сосуда находится  гелий массой , а в другой аргон массой . Средняя квадратичная скорость атомов аргона равна средней квадратичной скорости атомов гелия и составляет . Определите внутреннюю энергию  газа, оставшегося в той части сосуда, где первоначально находился гелий,  после установления равновесия в системе. Молярная масса гелия 0,004 кг/моль, а аргона  0,04 кг/моль. 

С 9

Теплоизолированный сосуд объемом  разделен пористой  перегородкой на 2 равные части. Атомы гелия могут свободно проникать через поры в перегородке, а атомы аргона - нет. В начальный момент в  одной части сосуда находится  гелий массой , а в другой аргон массой . Средняя квадратичная скорость атомов аргона равна средней квадратичной скорости атомов гелия и составляет . Определите внутреннюю энергию  гелий-аргоновой смеси после установления равновесия в системе. Молярная масса гелия 0,004 кг/моль, а аргона 0,04 кг/моль.


   Б

   А

 0 оС

 0 оС

100 оС

100 оС




1. mericrimes Геноцид экоцид психоцид как принципы доминирования Все тайны истории ~ Максим Акимов
2. по теме Группы и признаки обособленных второстепенных членов предложения
3. Доклады- 1Природные ресурсы Земли I
4. В Бельский Белорусская политехническая академия Минск В настоящее время становится все более очевидным
5. тема 7
6. Вятские Зори Город Вятские Поляны Кировской области
7. Школьные неврозы и причины их возникновения
8. Стресс жизни Причины стресса 4 1
9. Подготовка строительства Специфика управления строительством определяется особенностями строительн
10. Прекращение юридического лица при банкротстве
11. Социальная и этническая идентичности
12. Влияние философии развивающихся стран и стран с переходной экономикой на философию нового тысячелетия
13. варіантів відповідей правильним є тільки один
14. 1988 ученый с мировым именем автор] 200 книг и статей по программированию и информатике член британского обще
15. Лекция 38. Тонкий кишечник Тонкая кишка лат
16. на тему- А Фейгенбаум его работы по управлению качеством СТУДЕНТКА- ГРУППА- ПРОВЕР
17. Теоретические основы налогового регулирования банковской деятельности
18. Люди взрослые в большинстве своем на здоровье плюют и о том что едят задумываются не слишком- ну гастритик л
19. Вариант 30 Вопрос 1- Направление в теории познания XX в
20. Ислам - вера, образ жизни и закон Саудовской Аравии